SlideShare a Scribd company logo
Test Bank - Karch Focus on Nursing Pharmacology
9th Edition by Rebecca Tucker
Chapters 1 - 59
Table of Contents
Chapter 01- Introduction to Drugs
Chapter 02- Drugs and the Body
Chapter 03- Toxic Effects of Drugs
Chapter 04- The Nursing Process in Drug Therapy and Patient Safety
Chapter 05- Dosage Calculations
Chapter 06- Challenges to Effective Drug Therapy
Chapter 07- Introduction to Cell Physiology Chapter
08- Anti-infective Agents
Chapter 09- Antibiotics
Chapter 10- Antiviral Agents
Chapter 11- Antifungal Agents
Chapter 12- Antiprotozoal Agents
Chapter 13- Anthelmintic Agents
Chapter 14- Antineoplastic Agents
Chapter 15- Introduction to the Immune Response and Inflammation
Chapter 16- Anti-inflammatory, Antiarthritis, and Related Agents
Chapter 17- Immune Modulators
Chapter 18- Vaccines and Sera
Chapter 19- Introduction to Nerves and the Nervous System
Chapter 20- Anxiolytic and Hypnotic Agents
Chapter 21- Antidepressant Agents
Chapter 22- Psychotherapeutic Agents
Chapter 23- Antiseizure Agents Chapter
24- Antiparkinsonism Agents Chapter
25- Muscle Relaxants
Chapter 26- Narcotics, Narcotic Antagonists, and Antimigraine Agents
Chapter 27- General and Local Anesthetic Agents
Chapter 28- Neuromuscular Junction Blocking Agents Chapter
29- Introduction to the Autonomic Nervous System Chapter
30- Adrenergic Agonists
Chapter 31- Adrenergic Antagonists
Chapter 32- Cholinergic Agonists
Chapter 33- Anticholinergic Agents
Chapter 34- Introduction to the Endocrine System
Chapter 35- Hypothalamic and Pituitary Agents
Chapter 36- Adrenocortical Agents
Chapter 37- Thyroid and Parathyroid Agents Chapter
38- Agents to Control Blood Glucose Levels Chapter
39- Introduction to the Reproductive System
Chapter 40- Drugs Affecting the Female Reproductive System
Chapter 41- Drugs Affecting the Male Reproductive System
Chapter 42- Introduction to the Cardiovascular System Chapter
43- Drugs Affecting Blood Pressure
Chapter 44- Agents for Treating Heart Failure
Chapter 45- Antiarrhythmic Agents
Chapter 46- Antianginal Agents
Chapter 47- Lipid-Lowering Agents
Chapter 48- Drugs Affecting Blood Coagulation
Chapter 49- Drugs Used to Treat Anemias Chapter
50- Introduction to the Renal System Chapter 51-
Diuretic Agents
Chapter 52- Drugs Affecting the Urinary Tract and the Bladder Chapter
53- Introduction to the Respiratory System
Chapter 54- Drugs Acting on the Upper Respiratory Tract
Chapter 55- Drugs Acting on the Lower Respiratory Tract
Chapter 56- Introduction to the Gastrointestinal System
Chapter 57- Drugs Affecting Gastrointestinal Secretions
Chapter 58- Drugs Affecting Gastrointestinal Motility Chapter
59- Antiemetic Agents
Chapter 1: Introduction to Drugs
Cognitive Level: Application Difficulty:
Moderate
Integrated Process: Nursing process
1.A nurse working in radiology administers iodine to a patient who is having a
computerized axial tomography (CAT) scan. A nurse working on an oncology unit
administers chemotherapy to patients who have cancer. At the Public Health
Department, a nurse administers a measles-mumps-rubella (MMR) vaccine to a 14-
month-old child as a routine immunization. Which branch of pharmacology best
describes the actions of all three nurses?
A) Pharmacoeconomics
B) Pharmacotherapeutics
C) Pharmacodynamics
D) Pharmacokinetics
ANSWER: B
Response:
Pharmacology is the study of the biologic effects of chemicals. Nurses are involved with
clinical pharmacology or pharmacotherapeutics, which is a branch of pharmacology that
deals with the uses of drugs to treat, prevent, and diagnose disease. The nurse working
in radiology is administering a drug to help diagnose a disease. The nurse working on an
oncology unit is administering a drug to help treat a disease. Pharmacoeconomics
includes any costs involved in drug therapy.
Pharmacodynamics involves how a drug affects the body, and pharmacokinetics is how
the body acts on the drug.
Cognitive Level: Comprehension
Difficulty: Easy
Integrated Process: Nursing process
2.A physician has ordered intramuscular injections of morphine, a narcotic, every 4 hours
as needed for pain for a motor vehicle accident victim. The nurse is aware that there is a
high abuse potential for this drug; therefore, morphine is categorized as a:
A) Schedule I drug
B) Schedule II drug
C) Schedule III drug
D) Schedule IV drug
ANSWER: B
Response:
Narcotics such as morphine are considered schedule II drugs because of the high abuse
potential with severe dependence liability. Schedule I drugs have high abuse potential
and no accepted medical use. Schedule III drugs have a lesser abuse potential than
schedule II drugs and an accepted medical use. Schedule IV drugs have low abuse
potential and limited dependence liability.
Cognitive Level: Comprehension
Difficulty: Easy
Integrated Process: Communication and documentation
3.A nurse working for a drug company is involved in phase III drug evaluation studies.
Which of the following might the nurse be responsible for during this stage of drug
development?
Chapter 1: Introduction to Drugs
A) Working with animals who are given experimental drugs
B) Monitoring drug effects in patients who are selected to participate in a study,
who have the disease that the drug is meant to treat
C) Collecting records of symptoms that participants experience while taking a
drug and determining whether they are caused by the disease or the drug
D) Informing healthy, young volunteer participants of possible risks that could
occur from taking an experimental drug
ANSWER: C
Response:
Phase III studies involve use of a drug in a vast clinical market where patients are asked
to record any symptoms they experience while taking the drugs. Nurses may be
responsible for helping collect and analyze the information to be shared with the Food
and Drug Administration (FDA). Use of animals in drug testing is done in the
preclinical trials. A select group of patients who are involved in phase II studies
participate in studies where the participants have the disease the drug is intended to
treat. These patients are monitored closely for drug effects. Phase I studies involve
healthy human volunteers who are usually paid for their participation. Nurses may
observe for adverse effects and toxicity.
Cognitive Level: Comprehension Difficulty:
Moderate
Integrated Process: Nursing Process
4.Which of the following concepts is considered when generic drugs are substituted for
brand-name drugs?
A) Bioavailability
B) Critical concentration
C) Distribution
D) Half-life
ANSWER: A
Response:
Bioavailability is the portion of a dose of a drug that reaches the systemic circulation
and is available to act on body cells. Binders used in a generic drug may not be the same
as those used in the brand-name drug. Therefore, the way the body breaks down and
uses the drug may differ, which may eliminate a substitution. Critical
concentration is the amount of a drug that is needed to cause a therapeutic effect.
Distribution is the phase of pharmacokinetics that involves the movement of a drug to
the body's tissues. A drug's half-life is the time it takes for the amount of drug to decrease
to one-half of the peak level.
Cognitive Level: Analysis Difficulty:
Difficult
Integrated Process: Teaching/learning
5.A nurse is teaching her patient about the use of over-the-counter (OTC) drugs. Which of
the following statements best informs the patient about their safe use?
A) “OTC drugs are products that are available without prescription for self-
treatment of minor complaints.”
B) “OTC drugs are considered medications and should be reported on a drug
history.”
Chapter 1: Introduction to Drugs
C) “OTC drugs were approved as prescription drugs but later were found to be
safe without the need for a prescription.”
D) “OTC drugs need to be taken with caution. They can mask the signs and
symptoms of an underlying disease and interfere with prescription drug
therapy.”
ANSWER: D
Response:
OTC drugs are considered medications and should be reported. OTC drugs are
available without a prescription, although some were first approved as prescription
drugs. The most important teaching should relate to their safe use and that OTC drugs
can mask symptoms of disease and interfere with prescribed drugs.
Cognitive Level: Knowledge
Difficulty: Easy
Integrated Process: Nursing Process
6.Which of the following legislative acts allowed the Food and Drug Administration
(FDA) to tighten control over the quality of drugs and required that safety and efficacy
standards be established?
A) Pure Food and Drug Act of 1906
B) Federal Food, Drug, and Cosmetic Act of 1938
C) Durham Humphrey Amendment of 1951
D) Kefauver-Harris Act of 1962
ANSWER: D
Response:
The Kefauver-Harris Act was the result of the use of the 1960s drug thalidomide
(Thalomid). The public concern led to the legislation that gave the FDA regulatory
control over testing and evaluating of drugs and allowed it to set standards for efficacy
and safety. The Pure Food and Drug Act required labeling to eliminate false claims. The
Federal Food, Drug, and Cosmetic Act gave the FDA the power to enforce standards
for testing drug toxicity and monitoring labeling. The Durham- HumphreyAmendment
enforced prescriptions for distribution.
Cognitive Level: Application
Difficulty: Moderate
Integrated Process: Teaching/learning
7.A nurse is instructing a pregnant patient concerning the potential risk to her fetus
from a pregnancy category B drug. The nurse would inform the patient that:
A) “Adequate studies in pregnant women have demonstrated there is no risk to the
fetus.”
B) “Animal studies have not demonstrated a risk to the fetus but there have been
no adequate studies in pregnant women.”
C) “Animal studies have shown an adverse effect on the fetus but there are no
adequate studies in pregnant women.”
D) “There is evidence of human fetal risk but the potential benefits from use of the
drug may be acceptable despite potential risks.”
ANSWER: B
Response:
Category B indicates that animal studies have not demonstrated a risk to the fetus but
Chapter 1: Introduction to Drugs
there have been no adequate studies in pregnant women. However, there have not been
adequate studies in pregnant women to demonstrate risk to a fetus during the first
trimester of pregnancy and no evidence of risk in later trimesters. Category A indicates
that adequate studies in pregnant women have not demonstrated a risk to the fetus in
the first trimester or in later trimesters. Category C indicates that animal studies have
shown an adverse effect on the fetus but there have been no adequate studies in
humans. Category D reveals evidence of human fetal risk, but the potential benefits
from the use of the drugs in pregnant women may outweigh the risks.
Cognitive Level: Analysis Difficulty:
Difficult
Integrated Process: Teaching/learning
8. Discharge planning for patients leaving the hospital should include instructions on
the use of over-the-counter (OTC) drugs. Which comment by the patient would
demonstrate a good understanding of OTC drugs?
A) “OTC drugs are always safe and will not cause bad effects.”
B) “OTC drugs have been around for years and have not been tested by the Food
and Drug Administration (FDA).”
C) “OTC drugs are different from any drugs available by prescription and cost
less.”
D) “OTC drugs are thought to be safe when taken as directed.”
ANSWER: D
Response:
OTC drugs are drugs that have been determined to be safe when taken as directed.
They may have originally been prescription drugs that were tested by the FDA or they
may have been grandfathered in when the FDA laws changed. OTC education should
always be included as a part of the hospital discharge instructions.
Cognitive Level: Comprehension
Difficulty: Moderate
Integrated Process: Teaching/Learning
9. Which of the following would be the best source of drug information for a nurse?
A) Drug Facts and Comparisons book
B) A nurse's drug guide
C) A drug package insert
D) The Physician's Drug Reference (PDR)
ANSWER: B
Response:
The most user-friendly drug source for a nurse is a nurse's drug guidebook. A guide
provides nursing implications and patient teaching points that are most useful to
nurses. The Drug Facts and Comparisons book provides a wide range of drug
information but is hard to manipulate and is very expensive. A package insert
contains all of the chemical and drug company research information about a drug;
however, the information can be difficult to understand and the print is very small.
The PDR is heavily cross-referenced and difficult to use.
Cognitive Level: Comprehension
Chapter 1: Introduction to Drugs
Difficulty: Moderate
Integrated Process: Nursing Process
10. Which of the following statements best defines how a chemical becomes a drug?
A) A chemical must have a proven therapeutic value or efficacy without severe
toxicity or damaging properties to become a drug.
B) A chemical becomes a drug when it is introduced into the body to cause a
change.
C) A chemical is considered a drug when the Food and Drug Administration
(FDA) approves its release to be marketed.
D) A chemical must have demonstrated therapeutic value to become a drug.
ANSWER: A
Response:
A chemical must undergo a series of tests to determine its therapeutic value and
efficacy without severe toxicity or damaging properties before it is termed a drug.
Test results are reported to the FDA, which may or may not give approval.
Chapter 2: Drugs and the Body
Cognitive Level: Analysis Difficulty:
Difficult
Integrated Process: Nursing process
1. In which of the following patients would a nurse expect to see an alteration in drug
metabolism?
A) A 35-year-old female with cervical cancer
B) A 41-year-old male with kidney stones
C) A 50-year-old male with cirrhosis of the liver
D) A 62-year-old female in acute renal failure
ANSWER: C
Response:
The liver is the most important site of drug metabolism. If the liver is not functioning
effectively, as in patients with cirrhosis, drugs will not be metabolized normally and
toxic levels could develop. In cirrhosis patients the drug dosage will need to be altered
to ensure normal levels in the body.
Cognitive Level: Analysis Difficulty:
Difficult
Integrated Process: Nursing process
2.A patient has a drug level of 50 units/mL and the drug's half-life is 1 hour. If
concentrations above 25 units/mL are considered toxic and no more drug is given,
how long will it take for the blood level to reach the nontoxic range?
A) 30 minutes
B) 1 hour
C) 2 hours
D) 3 hours
ANSWER: C
Response:
Half-life is the time required for the serum concentration of a drug to decrease by 50%.
After 1 hour, the serum concentration would be 25 units/mL (50/2). After 2 hours, the
serum concentration would be 12.5 units/mL (25/2) and reach the nontoxic range.
Cognitive Level: Analysis Difficulty:
Difficult
Integrated Process: Nursing process
3.A nurse is caring for a patient who has recently moved from Vermont to south Florida.
The patient has been on the same antihypertensive drug for 6 years and has had stable
blood pressures and no side effects. Since her move, however, she reports having
“dizzy spells and weakness” and feels that the drug is no longer effective. The nurse
knows that one possible reason for the change in the effectiveness of the drug could be
which of the following?
A) The impact of the placebo effect on the patient's response
B) The accumulative effect of the drug if it has been taken for many years
C) The impact of the warmer environment on the patient's physical state
D) Problems with patient compliance with the drug regimen while on vacation
ANSWER: C
Response:
Chapter 2: Drugs and the Body
Antihypertensive drugs work to decrease the blood pressure. When a patient goes to a
climate that is much warmer than usual, blood vessels dilate and the blood pressure
falls. If a patient is taking an antihypertensive drug and enters a warmer climate, there is
a chance that the patient's blood pressure will drop too low, resulting in dizziness and
feelings of weakness. Most antihypertensives are metabolized and excreted and do not
accumulate in the body. Patients must be very compliant with their drug regimen while
on vacation. After several years on an antihypertensive drug, the effects of that drug are
known; therefore, the placebo effect should not be an issue.
Cognitive Level: Comprehension
Difficulty: Moderate
Integrated Process: Nursing Process
4. During which phase of pharmacokinetics could a significant drug-drug interaction
occur, resulting in one drug stimulating or blocking the metabolism of the other drug?
A) Absorption
B) Distribution
C) Biotransformation
D) Excretion
ANSWER: C
Response:
During biotransformation, a drug may be metabolized quicker if taken at the same time
as another drug, and higher doses of the drug will be needed to achieve the desired
effect. During absorption, one drug can prevent or accelerate absorption of the other
drug. During distribution, one drug competes for the protein-binding site of another
drug so the second drug cannot be transported to the reactive tissue. During excretion,
one drug competes for excretion with the other drug, leading to accumulation and toxic
effects of one of the drugs.
Cognitive Level: Comprehension
Difficulty: Moderate
Integrated Process: Nursing Process
5. Which statement accurately describes a drug agonist?
A) A drug that reacts with specific receptor sites on a cell and prevents the
reaction of another chemical with a different receptor site on that cell
B) A drug that interferes with the enzyme systems that act as catalysts for different
chemical reactions
C) A drug that interacts directly with receptor sites to cause the same activity that a
natural chemical would cause at that site
D) A drug that reacts with receptor sites to block normal stimulation, producing no
effect
ANSWER: C
Response:
Agonists are drugs that produce effects similar to those produced by naturally
occurring neurotransmitters, hormones, or other substances found in the body.
Noncompetitive antagonists are drugs that react with certain receptor sites, preventing
the reaction of another chemical with a different receptor site. Drug- enzyme
interactions interfere with the enzyme systems that stimulate various
Chapter 2: Drugs and the Body
chemical reactions.
Cognitive Level: Analysis
Difficulty: Difficult
Integrated Process: Nursing process
6.A nurse has been administering a drug to a patient intramuscularly (IM). The
physician discontinued the IM dose and wrote an order for the drug to be given
orally. The nurse notices that the oral dosage is considerably higher than the
parenteral dose and understands that this is due to:
A) Passive diffusion
B) Active transport
C) Glomerular filtration
D) First-pass effect
ANSWER: D
Response:
The first-pass effect involves drugs that are absorbed from the small intestine directly
into the portal venous system, which delivers the drug molecules to the liver. Once in
the liver, enzymes break the drug into metabolites, which may become active or may be
deactivated and readily excreted from the body. A large percentage of the oral dose is
usually destroyed and never reaches tissues. Oral dosages account for the phenomenon
to ensure an appropriate amount of the drug in the body to produce a therapeutic
action. Passive diffusion is the major process through which drugs are absorbed into
the body. Active transport is a process that uses energy to actively move a molecule
across a cell membrane and is often involved in drug excretion in the kidney.
Glomerular filtration is the passage of water and water-soluble components from the
plasma into the renal tubule.
Cognitive Level: Comprehension
Difficulty: Easy
Integrated Process: Nursing process
7.A nurse working as a member of a research team is exploring the unique differences in
drug response that each individual possesses based on his or her genetic makeup. This
area of study is called:
A) Pharmacotherapeutics
B) Pharmacodynamics
C) Pharmacoeconomics
D) Pharmacogenomics
ANSWER: D
Response:
Pharmacogenomics is the area of study that includes mapping of the human genome. In
the future, medical care and drug regimens may be personally designed based on a
person's unique genetic makeup. Pharmacotherapeutics is the branch of pharmacology
that deals with the use of drugs to treat, prevent, and diagnose disease.
Pharmacodynamics involves how a drug affects the body. Pharmacoeconomics includes
any costs involved in drug therapy.
Cognitive Level: Comprehension
Chapter 2: Drugs and the Body
Difficulty: Easy
Integrated Process: Nursing process
8.The nurse is aware that for a drug to work properly on reactive tissues, a sufficient
amount of the drug is needed to cause the therapeutic effect. The term used for this
amount is:
A) Critical concentration
B) Dynamic equilibrium
C) Selective toxicity
D) Active transport
ANSWER: A
Response:
A critical concentration of a drug must be present before a reaction occurs within our
cells. A dynamic equilibrium is obtained from absorption of a drug from the site of
drug entry, distribution to the active site, metabolism in the liver, and excretion from
the body to have a critical concentration. Selective toxicity is the ability of a drug to
attach only to those systems found in foreign cells. Active transport is the process that
uses energy to actively move a molecule across a cell membrane and is often involved in
drug excretion in the kidney.
Cognitive Level: Analysis
Difficulty: Difficult
Integrated Process: Nursing process
9.A nurse has received an order to administer two drugs at the same time of the day.
The nurse should first:
A) Wash her hands before handling the medications
B) Consult a drug guide for compatibility
C) Question the patient concerning drug allergies
D) Identify the patient by checking the armband and asking the patient to state his
name
ANSWER: B
Response:
A nurse should always first consult a drug guide for compatibility when two or more
drugs are being given at the same time. Once she is certain the drugs are compatible,
she should wash her hands and prepare the medication. She should identify the patient
appropriately and ask about drug allergies prior to administering the
medication.
Cognitive Level: Comprehension
Difficulty: Moderate
Integrated Process: Nursing Process
10.Which of the following factors affects absorption of intramuscular (IM)
administration of drugs?
A) Perfusion of blood to the subcutaneous tissue
B) Integrity of the mucous membranes
C) Environmental temperature
D) Blood flow to the gastrointestinal tract
ANSWER: C
Response:
Chapter 2: Drugs and the Body
A cold environmental temperature can cause muscles to vasoconstrict and decreases
absorption; a hot environment can cause vasodilation and increase absorption of IM
medications. Blood flow to the subcutaneous tissues interferes with subcutaneous
injection, and blood flow to the gastrointestinal tract causes alterations in absorption
for oral medications. The condition of mucous membranes can interfere with
sublingual (under the tongue) and buccal (in the cheek) administration of drugs.
Chapter 3: Toxic Effects of Drugs
Cognitive Level: Application Difficulty:
Difficult
Integrated Process: Teaching/learning
1.A nurse is instructing her patient about a newly prescribed drug. Which of the following
should be included in the teaching to help improve patient compliance and safety?
A) List of pharmacies where the drug can be obtained
B) Measures to alleviate any discomfort associated with adverse effects
C) The cost of the brand-name drug compared to the generic form
D) Statistics related to phase III of testing for the prescribed drug
ANSWER: B
Response:
If patients are aware of certain adverse effects and how to alleviate or decrease the
discomfort, they are more likely to continue taking the medication. A list of pharmacies
can be useful information but will not improve safety or compliance. Knowing the cost
of the brand-name versus the generic drug could also be helpful to the patient;
however, a substitution may not be allowable and the cost of a drug does not improve
patient safety. Most patients are not concerned with the statistics related to drug testing
and the results are therefore not shared with the patient.
Cognitive Level: Comprehension Difficulty:
Moderate
Integrated Process: Nursing process
2.A patient has been taking an oral antibiotic for 10 days. She reports to the nurse that
she has developed vaginal itching and a clear discharge. The nurse is aware that the
patient is experiencing which of the following?
A) An adverse reaction from the antibiotic
B) A stomatitis, which has caused an inflammation of the mucous membranes
C) An overdose of the drug that is damaging to more than one body system
D) A superinfection caused by the antibiotic, which has destroyed normal flora
ANSWER: D
Response:
Superinfections often occur with antibiotic use because the drug kills normal flora
bacteria. Vaginal itching and a clear discharge are signs and symptoms of a yeast
infection and not stomatitis. Vaginal itching and a clear discharge are not considered
adverse effects of an antibiotic. An overdose of a drug that damages more than one
body system is considered poisoning.
Cognitive Level: Analysis
Difficulty: Moderate
Integrated Process: Nursing process
3.A male patient is admitted to the emergency room in distress. The nurse suspects an
anaphylactic reaction from oral penicillin. Which assessment findings are important in
making this diagnosis?
A) Blood pressure: 186/100, difficulty breathing
B) Hematocrit (Hct): 32%, decreased urine output
C) Temperature: 102ºF, swollen joints
D) Profuse sweating, blood pressure: 92/58
Chapter 3: Toxic Effects of Drugs
ANSWER: A
Response:
An anaphylactic reaction is an immune reaction that causes a massive release of
histamine, resulting in edema and swelling, which can lead to respiratory distress and
increased blood pressure. A decreased Hct and decreased urine output suggest a
cytotoxic reaction. An increased temperature and swollen joints could suggest serum
sickness. Profuse sweating and a blood pressure of 92/58 may indicate cardiac- related
issues
Cognitive Level: Application
Difficulty: Moderate
Integrated Process: Teaching/learning
4.A patient is taking an antihistamine to relieve itchy, watery eyes and a runny nose
associated with seasonal allergies. A good nursing intervention would be to advise the
patient to:
A) Limit fluid intake to dry mucous membranes
B) Avoid driving or operating machinery
C) Report strange dreams or nightmares
D) Decrease dietary fat
ANSWER: B
Response:
An adverse effect of antihistamines is drowsiness, and injury can occur if the patient
drives or operates machinery. An increase in fluids would be indicated to help keep
nasal membranes moist. It is common for dreams to occur when taking medication and
it is not necessary to report them. Dietary fat should not interfere with the drug
metabolism of antihistamines.
Cognitive Level: Application
Difficulty: Difficult
Integrated Process: Teaching/learning
5.A nurse is teaching a group of five patients who are beginning drug therapy for AIDS.
Which of the following should be included in her instructions to the group?
A) “Take your medications as indicated. Poisoning can occur when an overdose
damages more than one body system.”
B) “Renal injury is common and can be caused by the first-pass effect.”
C) “A blood dyscrasia due to drug therapy can be serious. Call us if your skin
looks yellow.”
D) “Most drugs are metabolized in the liver. You should report dark red papules
immediately.”
ANSWER: A
Response:
Poisoning can lead to the potential for fatal reactions when more than one body
system is affected. Liver injury can be caused by the first-pass effect and can cause
the skin to have a yellow appearance. Most drugs are metabolized in the liver.
However, dark red papules appearing on extremities are characteristic of Stevens-
Johnson syndrome, a potentially fatal erythema multiforme exudativum.
Chapter 3: Toxic Effects of Drugs
Cognitive Level: Analysis
Difficulty: Difficult
Integrated Process: Nursing process
6.Potassium-sparing diuretics can lead to potassium retention. Which assessment
finding could indicate hyperkalemia?
A) Urine output of 1500 mL/24 hours
B) Blood pressure of 98/60
C) Potassium level of 5.9 mEq/L
D) Pulse: 60 and weak
ANSWER: C
Response:
The normal range of serum potassium for an adult is 3.5 to 5.0 mEq/L. A level higher
than 5.0 mEq/L can indicate hyperkalemia. Normal urinary output is between 1500 and
2000 mL per day. Urinary output below 1000 mL per day would include oliguria and
would indicate hyperkalemia. A decrease in blood pressure and pulse can indicate
hypokalemia.
Cognitive Level: Application
Difficulty: Moderate
Integrated Process: Teaching/learning
7.A nurse is talking with family members of an 80-year-old patient who is experiencing
tinnitus from taking a macrolide antibiotic. Which of the following should the nurse
include in her instructions regarding home care?
A) Keep the patient in the prone position when in bed.
B) Eliminate salt from the patient's diet.
C) Provide protective measures to prevent falling or injury.
D) Monitor exposure to sunlight.
ANSWER: C
Response:
Macrolide antibiotics can cause severe auditory nerve damage, which can cause
dizziness, ringing in the ears (tinnitus), and loss of balance and hearing. The patient
would be at high risk for injury due to falls. Usually a person who is dizzy is unable to
lie flat and needs to recline with the head elevated. Salt may be decreased but not
entirely eliminated from the diet. Sunlight is usually not an issue because clients who
experience tinnitus are inside their homes in a quiet environment or on bed rest.
Cognitive Level: Application Difficulty:
Moderate
Integrated Process: Nursing process
8.Alterations in glucose metabolism can occur from use of oral antidiabetic drugs. A
nurse will observe patients who are taking these drugs for which of the following?
A) Increased urination
B) Deep Kussmaul's respirations
C) Increased hunger
D) Tremors and lack of coordination
ANSWER: D
Response:
Lowering the blood glucose is an immediate problem for the nervous system, and
Chapter 3: Toxic Effects of Drugs
tremors and lack of coordination may be observed in the patient. When high blood
glucose levels occur, the patient may report increased hunger, urination, and thirst.
Prolonged high blood glucose levels may result in Kussmaul's respirations as the patient
tries to expel the excess CO2.
Cognitive Level: Analysis
Difficulty: Difficult
Integrated Process: Nursing process
9.A patient is being seen in the clinic for rheumatoid arthritis. Which of the following
statements would indicate that she may be developing a complication from her
rheumatoid medication, chloroquine (Aralen)?
A) “I have to urinate all the time.”
B) “Sometimes I have blurred vision.”
C) “I have tingling and numbness in my arms and legs.”
D) “Sometimes I feel like I am losing my balance when I walk.”
ANSWER: B
Response:
Chloroquine (Aralen) can cause ocular toxicity with blurring of vision, color vision
changes, corneal damage, and blindness. Increased urination and tingling and numbness
are signs of hyperkalemia and hypokalemia. Loss of balance can be caused by auditory
damage due to drug toxicity.
Cognitive Level: Analysis
Difficulty: Difficult
Integrated Process: Nursing process
10.A 68-year-old patient who must take antihistamines for severe allergies is planning a
vacation to Mexico. The nurse will encourage the patient to:
A) Avoid sightseeing during the hottest part of the day
B) Discontinue the antihistamines if he becomes extremely restless
C) Decrease the dosage of the drugs if he experiences excessive thirst
D) Continue taking the antihistamines even if he begins to hallucinate
ANSWER: A
Response:
Antihistamines can cause anticholinergic effects, which block the parasympathetic
nervous system by blocking cholinergic receptors, and decrease sweating, placing the
patient at high risk for heat stroke. Avoiding the hottest part of the day will help
prevent dehydration and heat prostration. Extreme restlessness could indicate
Parkinson-like syndrome not usually associated with antihistamines. Excessive thirst is
characteristic of hyperglycemia, and hallucinations are associated with drugs that affect
neurologic functioning. Nurses should never tell patients to decrease or discontinue a
drug unless the prescriber has instructed them to do so.
1. A 70-year-old patient has just received a drug, which can cause sedation. What would be
the priority nursing diagnosis for this patient?
A) Noncompliance: Cost of the drug
B) Deficient knowledge: Unfamiliar with drug therapy
C) Risk for injury: Related to adverse effects of the drug
D) Ineffective health maintenance: Need for medication
2. When providing drug therapy to a patient what is a responsibility of the nurse?
A) Teaching the patient how to cope with the effects of the drug to ensure the best
outcome
B) Helping the patient analyze the physiological and pathological effects of drugs
C) Warning the patient how most patients respond to the drug therapy
D) Encouraging the patient to increase or decrease dosages
3. You are gathering assessment data from a medication history of a 38-year-old male with
four children. What assessment information would be most important in providing care
for this patient?
A) The medication history of the patient's mother and/or father
B) The name of the patient's pharmacy
C) Insurance, financial support, and stability for the patient and his family
D) The last time the patient was hospitalized
4. During what phase of the nursing process would the nurse be required to consider the
efficacy of nursing interventions related to drug therapy?
A) Assessment
B) Nursing diagnosis
C) Interventions
D) Evaluation
5. Drug studies generally base the recommended adult dose of a drug needed to reach a
critical concentration on what measure?
A) A 60-year-old man
B) An average-sized woman
C) A 150-pound adult male
D) An obese adult
6. A nurse is caring for a child on the pediatric unit. A drug is ordered for the child, but
there is no pediatric dose listed for the drug. To make sure that the right dose has been
ordered, what will the nurse use to calculate the correct dose?
A) Surface area
B) Height
C) Birth date
D) Age at gestation
7. You are evaluating the discharge teaching you have done with your patient concerning
their drug therapy. What statement from the patient would indicate that teaching had
been effective?
A) “I have to take three pills each day. It doesn't matter when.”
B) “I should take the white pill in the morning and the green pill in the afternoon. I
don't know what they are called.”
C) “I need to tell all my other doctors that I am taking this Coumadin because it could
interact with other drugs.”
D) “I think I could cut that pill in half if I wanted to.”
8. The pharmacology students are learning about antibiotics that are known to cause
photosensitivity in some patients. What nursing measures would ensure the comfort of
patients taking these antibiotics and hopefully avoid serious adverse effects?
A) Avoiding the use of aspirin and other over-the counter pain products
B) Getting a wig or other head cover to avoid heat loss
C) Avoiding exposure to sunlight or use of a sunscreen and protective clothing if
exposure is inevitable
D) Providing frequent rest periods in a dark room
9. When taking a medication history on a patient why should the nurse ask about the use of
alternative therapies?
A) Patients who use new drugs are usually not compliant with medical regimens.
B) Many drug-alternative therapy interactions can cause serious problems and should
be avoided.
C) Natural products may be more effective and the drug may not be needed.
D) The cost of the drug and the alternative therapy may be too great for the patient to
handle.
10. Medication orders are written with very specific criteria and coincide with the 7 rights of
medication administration. What critical piece of information is missing from the
following medication order? Amoxicillin 250 mg every 8 hours.
A) Dose
B) Route
C) Frequency of administration
D) Trade name of the drug
11. You are the home health nurse caring for a 77-year-old patient with congestive heart
failure. When assessing this patient, in relation to their drug therapy, what is one thing
that would be important for you to include in your assessment?
A) Patient's age
B) Required lifestyle changes
C) Family members in the community
D) Compliance with nutritional recommendations
12. The student nurses are learning to weigh patients and do vital signs. Weighing a client is a
nursing intervention that is most important for what?
A) Dosage calculation
B) Assessing changes in fluid balance
C) Assessing changes in nutritional status
D) Caloric needs
13. Teaching the patient/caregiver about their medications is an important step in reducing
the risk of medication errors. What is an important teaching point about medications?
A) Speak up
B) Store medications in a warm humid place
C) Adjust your medication according to your weight
D) Keep a list of your prescribed medications
14. A 73-year-old male patient is being discharged home today. The discharge order for reads:
Take 10 mL of guaifenesin (Robitussin) PO q4h. The Robitussin comes in bottles
with plastic measuring caps. How much should the nurse teach the client to take at
home?
A) ½ tsp
B) 1 tsp
C) 2 tsp
D) 1 tbsp
15. It is important to evaluate the efficacy of what parameter when evaluating the drug
therapy of a patient?
A) Appropriateness of drug dosages
B) Compliance
C) Caregivers knowledge level
D) Nursing interventions
16. You are doing an admission assessment on a patient. During the medication history
portion of the assessment what would be important to assess with herbal supplements?
A) Research them for interactions with other medications
B) Discontinue them if taking prescription medications
C) Take them one hour before prescription medications
D) Take them three hours after prescription medications
17. A 27-year-old male is admitted to the Emergency Department (ED) after a serious
motorcycle accident. The patient has a head injury, abrasions covering the left side of his
body, a broken left femur, and internal injuries that are not fully assessed at this time. He
is transferred from the ED to the ICU. The nurse who is going to care for this patient in
the ICU knows that one of their most important responsibilities in regard to drug
therapy is what?
A) Support vital functions
B) Continue curative treatment
C) Institute life-saving treatment
D) Monitor the patient's response
18. When assessing a patient prior to starting a drug regimen it is often necessary to have a
baseline kidney function study done. What might kidney impairment indicate in relation to
drug therapy?
A) Pharmacodynamics stable at the therapeutic dose
B) Contraindication to the use of a drug
C) Increased dosage of a drug
D) Decreased adverse effects of a drug
19. A nurse is caring for a 77-year-old patient. The nurse would know that a normal
physiologic change that must be considered when planning drug therapy, and is
associated with aging is what?
A) Blood volume decreases
B) Subcutaneous tissue increases
C) Total body water increases
D) Muscle mass increases
20. You are caring for a patient who takes several drugs. You know that those patients most
likely to have adverse drug reactions are who?
A) Patients with coronary artery disease
B) Non-compliant patients
C) Patients who are on the recommended dose
D) Very young and very old patients
21. Which statement best describes drug efficacy/toxicity in pediatric clients?
A) Drug requirements for infants have been extensively studied.
B) Drug dosage is altered by age and weight in children.
C) Children always need smaller doses of medication than adults.
D) Infants and children are always at greater risk for drug toxicity with any
medication.
22. A 7-year-old male fell off a wood pile while playing. He has been admitted to the ICU
with multiple broken bones and internal bleeding. What should the nurse know about
drug therapy in this type of patient?
A) Adverse effects may be decreased
B) Therapeutic effects may be increased
C) Pharmacodynamics may be altered
D) Pharmacokinetics remain the same
23. You are designing a drug regimen for a patient. What should be considered?
A) Quality-of-life issues
B) Ethnicity of patient
C) Gender
D) Quantity-of-life issues
24. The nursing instructor is discussing drug therapy in the older adult. What would the
instructor tell the students could effect therapeutic dosing in an older adult?
A) In older adults, drugs have decreased GI absorption.
B) In older adults, drugs are released more quickly into circulation.
C) In older adults, drugs are distributed to a smaller portion of the tissue.
D) In older adults, drugs have an increased action.
25. In today's healthcare environment there is often more contact between the patient and
the nurse than between the patient and the physician. What does this make the nurse in
the best position to do?
A) Assess the patient's inability to document medication taken
B) Assess the patient's inability to communicate with caregivers
C) Assess the therapeutic success of the drug therapy
D) Assess the patient's reluctance to purchase the prescribed drug
26. You are discussing your patient's drug regimen with their primary care provider. The
patient has seen two specialists since seeing the primary care provider and been
diagnosed with renal impairment. Why would this be important information? (Mark all
that apply.)
A) To keep everyone in the loop
B) Help prevent medication errors
C) So the physician can answer family questions
D) Help prevent adverse effects
E) Help promote a safe and effective medication regimen
27. It is often necessary to obtain baseline data prior to initiating many forms of drug
therapy. These baseline data include what? (Mark all that apply.)
A) Education level
B) Allergies
C) Drug use
D) Number of members in family
E) Father's occupation
28. A 32-year-old female is admitted to your unit with a diagnosis of hypovolemia. You are
writing a care plan for this patient. You know that an appropriate nursing diagnosis to
help prevent medication errors is what?
A) Dysfunctional gastrointestinal motility
B) Ineffective self health maintenance
C) Risk for injury
D) Deficient fluid volume
29. When using the nursing process in medication therapy, what does it ensure?
A) That medications are given at the right time
B) That there is efficient and effective care
C) That the right dose of the drug is given to the patient
D) The right drug is given to the right patient at the right time
30. A 35-year-old male patient is admitted to the hospital with pneumonia. He was
originally being treated at home, but became worse when he quit taking his medicine.
What would be an appropriate nursing diagnoses for this patient?
A) Deficient knowledge: monitoring temperature
B) Noncompliance
C) Risk for injury related to office visits
D) Non-adherence: overuse
Answer Key
1. C
2. A
3. C
4. D
5. C
6. A
7. C
8. C
9. B
10. B
11. B
12. D
13. A
14. C
15. D
16. A
17. D
18. C
19. A
20. D
21. B
22. C
23. A
24. A
25. C
26. B, D, E
27. A, B, C
28. D
29. B
30. B
Chapter 5: Dosage Calculations
Cognitive Level: Knowledge Difficulty:
Easy
Integrated Process: Nursing process
1. Which of the following measuring systems used in drug preparation and delivery
uses the gram as the basic unit of solid measure?
A) Metric system
B) Apothecary system
C) Household system
D) Avoirdupois system
ANSWER: A
Response:
The metric system is the most widely used system of measurement and is based on the
decimal system. The gram is the basic unit of solid measure and the liter is the basic unit
of liquid measure. The apothecary system uses the grain as the basic unit of solid
measure. The household system uses the pound as the basic unit of measure.
The avoirdupois system uses ounces and grains but is mostly used by drug
manufacturers for bulk medications.
Cognitive Level: Analysis Difficulty:
Difficult
Integrated Process: Nursing process
2.A nurse is explaining to a young mother why she cannot give her 2-year-old child an
adult dose of Tylenol. Which of the following statements would indicate that the mother
needs further education?
A) “There could be a time when my child may need a higher dose than normal.”
B) “My baby's dose of Tylenol is based on a healthy adult male.”
C) “My baby can't handle a high dose of Tylenol because her liver may be
damaged.”
D) “My child's dose of Tylenol should be based on her weight or age.”
ANSWER: B
Response:
A child's dose is never based on an adult's dose. However, on rare occasions a child's
dose might be higher than normal if a critical concentration cannot be reached with a
smaller dose and a higher dose would not be harmful. Benefits from the increased
dosage would have to outweigh the risk for adverse or toxic effects. A child's organs
may not be mature enough to handle drugs, causing drug metabolism to be altered. A
child's dosages are determined by the age, weight, or body surface.
Cognitive Level: Comprehension
Difficulty: Easy
Integrated Process: Nursing process
3.A nurse is calculating a child's medication dose by using Clark's Rule. Which of the
following calculations represents the application of Clark's Rule?
A) Infant's age in months/150 months × the average adult dose
B) Child's age in years/child's age in years + 12 times the average adult dose
C) Weight of child in pounds/150 pounds × the average adult dose
D) Surface area in square meters/1.73 × the average adult dose
ANSWER: C
Chapter 5: Dosage Calculations
Response:
Clark's Rule uses the child's weight to calculate the dose and assumes the adult dose is
based on a 150-pound person. Fried's Rule applies to a child younger than 1 year of age
and assumes than an adult dose would be appropriate for a child who is 12.5 years (150
months) old. Young's Rule applies to children 1 to 12 years of age. Surface area
calculation of a child's dose is determined with the use of a nomogram including the
child's height and weight.
Cognitive Level: Application Difficulty:
Moderate
Integrated Process: Nursing process
4.A patient is to receive a daily dose of 240 mg of a medication. The dose is supposed to
be divided evenly so that the drug is given every 6 hours. How much of the drug
should be given at each dose?
A) 30 mg
B) 40 mg
C) 50 mg
D) 60 mg
ANSWER: D
Response:
Because there are 24 hours in a day, giving a drug every 6 hours would mean giving the
drug four times a day. Because the total daily dose is 240 mg, dividing that dose by
four would mean each dose should be 60 mg.
Cognitive Level: Application Difficulty:
Moderate
Integrated Process: Nursing process
5.A physician order is written for a patient to receive 500 mL of normal saline IV over 8
hours. If the IV infusion set delivers 15 drops/mL, how many drops per minute should
the patient receive?
A) 15 drops/min
B) 20 drops/min
C) 32 drops/min
D) 64 drops/min
ANSWER: A
Response:
If a patient were to receive 500 mL in 8 hours, dividing 500 by 8 would mean that the
patient would receive 62.5 mL in 1 hour, or 60 minutes. Setting up the equation, 15
drops/mL/X = 62.5 mL/60 minutes, and cross-multiplying, the answer will be 15
drops/min.
Cognitive Level: Application
Difficulty: Moderate
Integrated Process: Nursing process
6.Thirty-two units of Humulin insulin is ordered for the patient. The insulin is supplied in
a multidose vial that is labeled 100 units/mL. How much insulin would be needed to
treat this patient?
Chapter 5: Dosage Calculations
A) 0.032 cc
B) 0.32 cc
C) 3.2 cc
D) 32 cc
ANSWER: B
Response:
A cc is equivalent to an mL. There are 100 units in each cc. Divide that amount by 32
units for the answer (0.32 cc).
Cognitive Level: Application Difficulty:
Difficult
Integrated Process: Nursing process
7.A young adult is receiving a maintenance dose of aminophylline, 3 mg/kg PO q6h. The
patient weighs 50 kg. How much should the patient receive in a 24-hour period?
A) 200 mg
B) 400 mg
C) 600 mg
D) 800 mg
ANSWER: C
Response:
Use the formula, amount of drug prescribed × weight of the patient in kg × 4 (q6h in a
24-hour period = 4). 3 × 50 = 150 mg to be given every 6 hours. To determine how
much the patient will receive in a 24-hour period, divide 6 into 24 to equal 4 doses. 4
× 150 = 600 mg.
Cognitive Level: Application Difficulty:
Moderate
Integrated Process: Nursing process
8. An order is written for oxazepam for a 6-year-old child. The nurse notices that there is
no established dosage for children 6 to 12 years of age. Knowing that the usual adult
dose is 10 mg t.i.d., what would the nurse calculate the appropriate dose to be?
A) 0.03 mg t.i.d.
B) 0.3 mg t.i.d.
C) 1.8 mg t.i.d.
D) 3.3 mg t.i.d.
ANSWER: D
Response:
Because the nurse only knows the child's age, the nurse would need to use Young's
Rule to determine the appropriate dosage. Use the formula, child's dose equals the
age of the child in years divided by the child's age plus 12 times the average adult
dose to calculate the answer. Dose = (6/[6 + 12]) × 10 mg (6/18 = 0.33 × 10 = 3.3).
Cognitive Level: Application Difficulty:
Moderate
Integrated Process: Nursing process
9. An order is written for a patient to receive 1000 mL of normal saline IV over 8 hours. If
the IV infusion set is a microdrip set that delivers 60 drops/mL, how many drops
Chapter 5: Dosage Calculations
per minute should the patient be receiving?
A) 60 drops/min
B) 120 drops/min
C) 240 drops/min
D) 480
drops/min ANSWER:B
Response:
If a patient were to receive 1000 mL in 8 hours, dividing 1000 by 8 would mean that
the patient would receive 125 mL in 1 hour, or 60 minutes. Setting up the equation, 60
drops/mL × X = 125 mL/60 min, and cross-multiplying, the answer is 120
drops/min.
Cognitive Level: Application Difficulty:
Moderate
Integrated Process: Nursing process
10.A patient needs to take 1 g PO of cefadroxil. The drug comes in 500-mg tablets. How
many tablets should the patient take?
A) 0.5 tablet
B) 1 tablet
C) 2 tablets
D) 3 tablets
ANSWER: C
Response:
Convert 1 g to mg by multiplying 1 g times 1000 mg. There are 500 mg in each tablet.
Dividing the 1000-mg prescribed dosage by the 500-mg available dosage, the answer is
2 tablets.
1.A nurse is preparing a prescribed dose of rabeprazole for a client with gastric ulcers. The
nursing drug guide does not list treatment of gastric ulcers among the recognized indications
for this drug. What should the nurse do?
A) Administer the drug as ordered
B) Clarify with the prescriber concerning the ordered drug
C) Document this potential error in the client's health record
D) Ask the client whether he or she has taken this drug before
2.The nurse is applying the Centers for Disease Control and Prevention (CDC)
recommendations for the possibility of bioterrorism. What action should the nurse
perform?
A) Post updated information on signs and symptoms of infections caused by biological
agents
B) Provide guidelines for treating clients exposed to, or potentially exposed to,
biological agents
C) Remain current on recognition and treatment of infections caused by biological
weapons
D) Advocate for increased funding for research involving bioterrorism and client
treatment
3.A nurse's learning goals for the year include reviewing the most up-to-date information about
emergency preparedness related to bioterrorism agents. What should the nurse do?
A) Read textbooks devoted to the topic
B) Review best practices around triage and emergency care
C) Read journal articles about bioterrorism agents
D) Visit the Centers for Disease Control and Prevention (CDC) Web site
4.The nurse is assessing a client with diabetes who reports several hypoglycemic episodes
during the past 3 weeks. What assessment question best addresses a potential causative
factor?
A) “Do you ever take St. John's wort?”
B) “Have you used any recreational drugs in the past few weeks?”
Chapter 5: Dosage Calculations
IF YOU WANT THIS TEST BANK OR SOLUTION
MANUAL EMAIL ME kevinkariuki227@gmail.com TO
RECEIVE ALL CHAPTERS IN PDF FORMAT
IF YOU WANT THIS TEST BANK OR SOLUTION
MANUAL EMAIL ME kevinkariuki227@gmail.com TO
RECEIVE ALL CHAPTERS IN PDF FORMAT

More Related Content

Similar to Test bank for focus on nursing pharmacology 7th edition by amy m karch.pdf

How to recognize ADRs in patients.@ Clinical Pharmacy
How to recognize ADRs in patients.@ Clinical PharmacyHow to recognize ADRs in patients.@ Clinical Pharmacy
How to recognize ADRs in patients.@ Clinical Pharmacy
Drpradeepthi
 
Hospital pharmacy
Hospital pharmacyHospital pharmacy
Hospital pharmacy
Fariha Jerin
 
HISTORY, CONCEPT AND ITS IMPORTANCE IN DRUG DEVELOPMENT.pptx
HISTORY, CONCEPT AND ITS IMPORTANCE IN DRUG DEVELOPMENT.pptxHISTORY, CONCEPT AND ITS IMPORTANCE IN DRUG DEVELOPMENT.pptx
HISTORY, CONCEPT AND ITS IMPORTANCE IN DRUG DEVELOPMENT.pptx
Dhanashri Prakash Sonavane
 
A Study on Knowledge of Pharmacist on Risk of Medication Use During Pregnancy...
A Study on Knowledge of Pharmacist on Risk of Medication Use During Pregnancy...A Study on Knowledge of Pharmacist on Risk of Medication Use During Pregnancy...
A Study on Knowledge of Pharmacist on Risk of Medication Use During Pregnancy...
Sunil Vadithya
 
Pharmacovigilance & Adverse drug reaction
Pharmacovigilance & Adverse drug reactionPharmacovigilance & Adverse drug reaction
Pharmacovigilance & Adverse drug reaction
Rahul Bhati
 
The paradigm of drug therapy
The paradigm of drug therapyThe paradigm of drug therapy
The paradigm of drug therapy
Eugene Shorikov
 
Phase 1 clinical trial
Phase 1 clinical trialPhase 1 clinical trial
Phase 1 clinical trial
Banhisikha Adhikari
 
The Evolution of Forensic Pharmacovigilance in Healthcare.pptx
The Evolution of Forensic Pharmacovigilance in Healthcare.pptxThe Evolution of Forensic Pharmacovigilance in Healthcare.pptx
The Evolution of Forensic Pharmacovigilance in Healthcare.pptx
Drhaneef2
 
Orientation To Pharmacology
Orientation To PharmacologyOrientation To Pharmacology
Orientation To PharmacologyTpetrici
 
Reduction of harm from high risk medications
Reduction of harm from high risk medicationsReduction of harm from high risk medications
Reduction of harm from high risk medications
Apollo Hospitals
 
pharmaco pharmaco-epidemiology
pharmaco pharmaco-epidemiologypharmaco pharmaco-epidemiology
pharmaco pharmaco-epidemiology
farranajwa
 
Pharmacovigilance overview
Pharmacovigilance overviewPharmacovigilance overview
Pharmacovigilance overviewSunil Boreddy Rx
 
Regulatory Requirements For New Drug Approval
Regulatory Requirements For New Drug ApprovalRegulatory Requirements For New Drug Approval
Regulatory Requirements For New Drug Approval
Shagufta Farooqui
 
Pharmacovigilance and adr
Pharmacovigilance and adrPharmacovigilance and adr
Pharmacovigilance and adr
Nilesh Siddhawar
 
clinical pharmacy
clinical pharmacyclinical pharmacy
clinical pharmacy
Sohan Patel
 
Basic aspects of Pharmacovigilance (Clinical Research & Pharmacovigilance).pptx
Basic aspects of Pharmacovigilance (Clinical Research & Pharmacovigilance).pptxBasic aspects of Pharmacovigilance (Clinical Research & Pharmacovigilance).pptx
Basic aspects of Pharmacovigilance (Clinical Research & Pharmacovigilance).pptx
Dureshahwar khan
 
Introduction to pharmacovigilance
Introduction to pharmacovigilanceIntroduction to pharmacovigilance
Introduction to pharmacovigilanceNahla Amin
 
1591115199267_Pharmacovigilance.pptx
1591115199267_Pharmacovigilance.pptx1591115199267_Pharmacovigilance.pptx
1591115199267_Pharmacovigilance.pptx
DrAniqaSundas
 
Phase clinicaltrial
Phase clinicaltrialPhase clinicaltrial
Phase clinicaltrial
lillibabu
 

Similar to Test bank for focus on nursing pharmacology 7th edition by amy m karch.pdf (20)

How to recognize ADRs in patients.@ Clinical Pharmacy
How to recognize ADRs in patients.@ Clinical PharmacyHow to recognize ADRs in patients.@ Clinical Pharmacy
How to recognize ADRs in patients.@ Clinical Pharmacy
 
Hospital pharmacy
Hospital pharmacyHospital pharmacy
Hospital pharmacy
 
HISTORY, CONCEPT AND ITS IMPORTANCE IN DRUG DEVELOPMENT.pptx
HISTORY, CONCEPT AND ITS IMPORTANCE IN DRUG DEVELOPMENT.pptxHISTORY, CONCEPT AND ITS IMPORTANCE IN DRUG DEVELOPMENT.pptx
HISTORY, CONCEPT AND ITS IMPORTANCE IN DRUG DEVELOPMENT.pptx
 
A Study on Knowledge of Pharmacist on Risk of Medication Use During Pregnancy...
A Study on Knowledge of Pharmacist on Risk of Medication Use During Pregnancy...A Study on Knowledge of Pharmacist on Risk of Medication Use During Pregnancy...
A Study on Knowledge of Pharmacist on Risk of Medication Use During Pregnancy...
 
Pharmacovigilance & Adverse drug reaction
Pharmacovigilance & Adverse drug reactionPharmacovigilance & Adverse drug reaction
Pharmacovigilance & Adverse drug reaction
 
Mine...
Mine...Mine...
Mine...
 
The paradigm of drug therapy
The paradigm of drug therapyThe paradigm of drug therapy
The paradigm of drug therapy
 
Phase 1 clinical trial
Phase 1 clinical trialPhase 1 clinical trial
Phase 1 clinical trial
 
The Evolution of Forensic Pharmacovigilance in Healthcare.pptx
The Evolution of Forensic Pharmacovigilance in Healthcare.pptxThe Evolution of Forensic Pharmacovigilance in Healthcare.pptx
The Evolution of Forensic Pharmacovigilance in Healthcare.pptx
 
Orientation To Pharmacology
Orientation To PharmacologyOrientation To Pharmacology
Orientation To Pharmacology
 
Reduction of harm from high risk medications
Reduction of harm from high risk medicationsReduction of harm from high risk medications
Reduction of harm from high risk medications
 
pharmaco pharmaco-epidemiology
pharmaco pharmaco-epidemiologypharmaco pharmaco-epidemiology
pharmaco pharmaco-epidemiology
 
Pharmacovigilance overview
Pharmacovigilance overviewPharmacovigilance overview
Pharmacovigilance overview
 
Regulatory Requirements For New Drug Approval
Regulatory Requirements For New Drug ApprovalRegulatory Requirements For New Drug Approval
Regulatory Requirements For New Drug Approval
 
Pharmacovigilance and adr
Pharmacovigilance and adrPharmacovigilance and adr
Pharmacovigilance and adr
 
clinical pharmacy
clinical pharmacyclinical pharmacy
clinical pharmacy
 
Basic aspects of Pharmacovigilance (Clinical Research & Pharmacovigilance).pptx
Basic aspects of Pharmacovigilance (Clinical Research & Pharmacovigilance).pptxBasic aspects of Pharmacovigilance (Clinical Research & Pharmacovigilance).pptx
Basic aspects of Pharmacovigilance (Clinical Research & Pharmacovigilance).pptx
 
Introduction to pharmacovigilance
Introduction to pharmacovigilanceIntroduction to pharmacovigilance
Introduction to pharmacovigilance
 
1591115199267_Pharmacovigilance.pptx
1591115199267_Pharmacovigilance.pptx1591115199267_Pharmacovigilance.pptx
1591115199267_Pharmacovigilance.pptx
 
Phase clinicaltrial
Phase clinicaltrialPhase clinicaltrial
Phase clinicaltrial
 

More from robinsonayot

Test bank for discovering the life span 4th edition robert s feldman (1).pdf
Test bank for discovering the life span 4th edition robert s feldman (1).pdfTest bank for discovering the life span 4th edition robert s feldman (1).pdf
Test bank for discovering the life span 4th edition robert s feldman (1).pdf
robinsonayot
 
Test bank for discovering the life span 4th edition robert s feldman.pdf
Test bank for discovering the life span 4th edition robert s feldman.pdfTest bank for discovering the life span 4th edition robert s feldman.pdf
Test bank for discovering the life span 4th edition robert s feldman.pdf
robinsonayot
 
TEST BANK FOR ESSENTIALS OF NURSING LEADERSHIP AND MANAGEMENT, 7TH EDITION.pdf
TEST BANK FOR ESSENTIALS OF NURSING LEADERSHIP AND MANAGEMENT, 7TH EDITION.pdfTEST BANK FOR ESSENTIALS OF NURSING LEADERSHIP AND MANAGEMENT, 7TH EDITION.pdf
TEST BANK FOR ESSENTIALS OF NURSING LEADERSHIP AND MANAGEMENT, 7TH EDITION.pdf
robinsonayot
 
Test bank for davis advantage for pathophysiology 2nd edition by caprio.pdf
Test bank for davis advantage for pathophysiology 2nd edition by caprio.pdfTest bank for davis advantage for pathophysiology 2nd edition by caprio.pdf
Test bank for davis advantage for pathophysiology 2nd edition by caprio.pdf
robinsonayot
 
Test bank for current medical diagnosis and treatment 2023 2024 62nd edition ...
Test bank for current medical diagnosis and treatment 2023 2024 62nd edition ...Test bank for current medical diagnosis and treatment 2023 2024 62nd edition ...
Test bank for current medical diagnosis and treatment 2023 2024 62nd edition ...
robinsonayot
 
Test bank for current diagnosis and treatment pediatrics twenty fourth editio...
Test bank for current diagnosis and treatment pediatrics twenty fourth editio...Test bank for current diagnosis and treatment pediatrics twenty fourth editio...
Test bank for current diagnosis and treatment pediatrics twenty fourth editio...
robinsonayot
 
Test bank for critical care nursing a holistic approach 11th edition morton f...
Test bank for critical care nursing a holistic approach 11th edition morton f...Test bank for critical care nursing a holistic approach 11th edition morton f...
Test bank for critical care nursing a holistic approach 11th edition morton f...
robinsonayot
 
Test bank for consumer behaviour buying having and being eighth canadian edit...
Test bank for consumer behaviour buying having and being eighth canadian edit...Test bank for consumer behaviour buying having and being eighth canadian edit...
Test bank for consumer behaviour buying having and being eighth canadian edit...
robinsonayot
 
Test Bank For Comprehensive Radiographic Pathology 7th Edition By Eisenberg.pdf
Test Bank For Comprehensive Radiographic Pathology 7th Edition By Eisenberg.pdfTest Bank For Comprehensive Radiographic Pathology 7th Edition By Eisenberg.pdf
Test Bank For Comprehensive Radiographic Pathology 7th Edition By Eisenberg.pdf
robinsonayot
 
Test bank for community public health nursing evidence for practice 4TH editi...
Test bank for community public health nursing evidence for practice 4TH editi...Test bank for community public health nursing evidence for practice 4TH editi...
Test bank for community public health nursing evidence for practice 4TH editi...
robinsonayot
 
Test bank for clinical procedures for medical assistants 10th edition by bone...
Test bank for clinical procedures for medical assistants 10th edition by bone...Test bank for clinical procedures for medical assistants 10th edition by bone...
Test bank for clinical procedures for medical assistants 10th edition by bone...
robinsonayot
 
Test bank for community health nursing a canadian perspective 5th edition by ...
Test bank for community health nursing a canadian perspective 5th edition by ...Test bank for community health nursing a canadian perspective 5th edition by ...
Test bank for community health nursing a canadian perspective 5th edition by ...
robinsonayot
 
Test bank for beckmann and ling s obstetrics and gynecology 8th edition by ro...
Test bank for beckmann and ling s obstetrics and gynecology 8th edition by ro...Test bank for beckmann and ling s obstetrics and gynecology 8th edition by ro...
Test bank for beckmann and ling s obstetrics and gynecology 8th edition by ro...
robinsonayot
 
Test Bank for Anatomy of Oriented Structure 8th edition.pdf
Test Bank for Anatomy of Oriented Structure 8th edition.pdfTest Bank for Anatomy of Oriented Structure 8th edition.pdf
Test Bank for Anatomy of Oriented Structure 8th edition.pdf
robinsonayot
 
Test bank for advanced assessment interpreting findings and formulating diffe...
Test bank for advanced assessment interpreting findings and formulating diffe...Test bank for advanced assessment interpreting findings and formulating diffe...
Test bank for advanced assessment interpreting findings and formulating diffe...
robinsonayot
 
Test bank for advanced assessment interpreting findings and formulating diffe...
Test bank for advanced assessment interpreting findings and formulating diffe...Test bank for advanced assessment interpreting findings and formulating diffe...
Test bank for advanced assessment interpreting findings and formulating diffe...
robinsonayot
 
TEST BANK Essentials of dental radiography 9th edition by Evelyn Thomson, Orl...
TEST BANK Essentials of dental radiography 9th edition by Evelyn Thomson, Orl...TEST BANK Essentials of dental radiography 9th edition by Evelyn Thomson, Orl...
TEST BANK Essentials of dental radiography 9th edition by Evelyn Thomson, Orl...
robinsonayot
 
Test bank calculating drug dosages a patient safe approach to nursing and mat...
Test bank calculating drug dosages a patient safe approach to nursing and mat...Test bank calculating drug dosages a patient safe approach to nursing and mat...
Test bank calculating drug dosages a patient safe approach to nursing and mat...
robinsonayot
 
Test bank advanced health assessment and differential diagnosis essentials fo...
Test bank advanced health assessment and differential diagnosis essentials fo...Test bank advanced health assessment and differential diagnosis essentials fo...
Test bank advanced health assessment and differential diagnosis essentials fo...
robinsonayot
 
TEST BANK For Family Practice Guidelines, 5th Edition by Jill C. Cash; Cheryl...
TEST BANK For Family Practice Guidelines, 5th Edition by Jill C. Cash; Cheryl...TEST BANK For Family Practice Guidelines, 5th Edition by Jill C. Cash; Cheryl...
TEST BANK For Family Practice Guidelines, 5th Edition by Jill C. Cash; Cheryl...
robinsonayot
 

More from robinsonayot (20)

Test bank for discovering the life span 4th edition robert s feldman (1).pdf
Test bank for discovering the life span 4th edition robert s feldman (1).pdfTest bank for discovering the life span 4th edition robert s feldman (1).pdf
Test bank for discovering the life span 4th edition robert s feldman (1).pdf
 
Test bank for discovering the life span 4th edition robert s feldman.pdf
Test bank for discovering the life span 4th edition robert s feldman.pdfTest bank for discovering the life span 4th edition robert s feldman.pdf
Test bank for discovering the life span 4th edition robert s feldman.pdf
 
TEST BANK FOR ESSENTIALS OF NURSING LEADERSHIP AND MANAGEMENT, 7TH EDITION.pdf
TEST BANK FOR ESSENTIALS OF NURSING LEADERSHIP AND MANAGEMENT, 7TH EDITION.pdfTEST BANK FOR ESSENTIALS OF NURSING LEADERSHIP AND MANAGEMENT, 7TH EDITION.pdf
TEST BANK FOR ESSENTIALS OF NURSING LEADERSHIP AND MANAGEMENT, 7TH EDITION.pdf
 
Test bank for davis advantage for pathophysiology 2nd edition by caprio.pdf
Test bank for davis advantage for pathophysiology 2nd edition by caprio.pdfTest bank for davis advantage for pathophysiology 2nd edition by caprio.pdf
Test bank for davis advantage for pathophysiology 2nd edition by caprio.pdf
 
Test bank for current medical diagnosis and treatment 2023 2024 62nd edition ...
Test bank for current medical diagnosis and treatment 2023 2024 62nd edition ...Test bank for current medical diagnosis and treatment 2023 2024 62nd edition ...
Test bank for current medical diagnosis and treatment 2023 2024 62nd edition ...
 
Test bank for current diagnosis and treatment pediatrics twenty fourth editio...
Test bank for current diagnosis and treatment pediatrics twenty fourth editio...Test bank for current diagnosis and treatment pediatrics twenty fourth editio...
Test bank for current diagnosis and treatment pediatrics twenty fourth editio...
 
Test bank for critical care nursing a holistic approach 11th edition morton f...
Test bank for critical care nursing a holistic approach 11th edition morton f...Test bank for critical care nursing a holistic approach 11th edition morton f...
Test bank for critical care nursing a holistic approach 11th edition morton f...
 
Test bank for consumer behaviour buying having and being eighth canadian edit...
Test bank for consumer behaviour buying having and being eighth canadian edit...Test bank for consumer behaviour buying having and being eighth canadian edit...
Test bank for consumer behaviour buying having and being eighth canadian edit...
 
Test Bank For Comprehensive Radiographic Pathology 7th Edition By Eisenberg.pdf
Test Bank For Comprehensive Radiographic Pathology 7th Edition By Eisenberg.pdfTest Bank For Comprehensive Radiographic Pathology 7th Edition By Eisenberg.pdf
Test Bank For Comprehensive Radiographic Pathology 7th Edition By Eisenberg.pdf
 
Test bank for community public health nursing evidence for practice 4TH editi...
Test bank for community public health nursing evidence for practice 4TH editi...Test bank for community public health nursing evidence for practice 4TH editi...
Test bank for community public health nursing evidence for practice 4TH editi...
 
Test bank for clinical procedures for medical assistants 10th edition by bone...
Test bank for clinical procedures for medical assistants 10th edition by bone...Test bank for clinical procedures for medical assistants 10th edition by bone...
Test bank for clinical procedures for medical assistants 10th edition by bone...
 
Test bank for community health nursing a canadian perspective 5th edition by ...
Test bank for community health nursing a canadian perspective 5th edition by ...Test bank for community health nursing a canadian perspective 5th edition by ...
Test bank for community health nursing a canadian perspective 5th edition by ...
 
Test bank for beckmann and ling s obstetrics and gynecology 8th edition by ro...
Test bank for beckmann and ling s obstetrics and gynecology 8th edition by ro...Test bank for beckmann and ling s obstetrics and gynecology 8th edition by ro...
Test bank for beckmann and ling s obstetrics and gynecology 8th edition by ro...
 
Test Bank for Anatomy of Oriented Structure 8th edition.pdf
Test Bank for Anatomy of Oriented Structure 8th edition.pdfTest Bank for Anatomy of Oriented Structure 8th edition.pdf
Test Bank for Anatomy of Oriented Structure 8th edition.pdf
 
Test bank for advanced assessment interpreting findings and formulating diffe...
Test bank for advanced assessment interpreting findings and formulating diffe...Test bank for advanced assessment interpreting findings and formulating diffe...
Test bank for advanced assessment interpreting findings and formulating diffe...
 
Test bank for advanced assessment interpreting findings and formulating diffe...
Test bank for advanced assessment interpreting findings and formulating diffe...Test bank for advanced assessment interpreting findings and formulating diffe...
Test bank for advanced assessment interpreting findings and formulating diffe...
 
TEST BANK Essentials of dental radiography 9th edition by Evelyn Thomson, Orl...
TEST BANK Essentials of dental radiography 9th edition by Evelyn Thomson, Orl...TEST BANK Essentials of dental radiography 9th edition by Evelyn Thomson, Orl...
TEST BANK Essentials of dental radiography 9th edition by Evelyn Thomson, Orl...
 
Test bank calculating drug dosages a patient safe approach to nursing and mat...
Test bank calculating drug dosages a patient safe approach to nursing and mat...Test bank calculating drug dosages a patient safe approach to nursing and mat...
Test bank calculating drug dosages a patient safe approach to nursing and mat...
 
Test bank advanced health assessment and differential diagnosis essentials fo...
Test bank advanced health assessment and differential diagnosis essentials fo...Test bank advanced health assessment and differential diagnosis essentials fo...
Test bank advanced health assessment and differential diagnosis essentials fo...
 
TEST BANK For Family Practice Guidelines, 5th Edition by Jill C. Cash; Cheryl...
TEST BANK For Family Practice Guidelines, 5th Edition by Jill C. Cash; Cheryl...TEST BANK For Family Practice Guidelines, 5th Edition by Jill C. Cash; Cheryl...
TEST BANK For Family Practice Guidelines, 5th Edition by Jill C. Cash; Cheryl...
 

Recently uploaded

Flu Vaccine Alert in Bangalore Karnataka
Flu Vaccine Alert in Bangalore KarnatakaFlu Vaccine Alert in Bangalore Karnataka
Flu Vaccine Alert in Bangalore Karnataka
addon Scans
 
Cervical & Brachial Plexus By Dr. RIG.pptx
Cervical & Brachial Plexus By Dr. RIG.pptxCervical & Brachial Plexus By Dr. RIG.pptx
Cervical & Brachial Plexus By Dr. RIG.pptx
Dr. Rabia Inam Gandapore
 
Antiulcer drugs Advance Pharmacology .pptx
Antiulcer drugs Advance Pharmacology .pptxAntiulcer drugs Advance Pharmacology .pptx
Antiulcer drugs Advance Pharmacology .pptx
Rohit chaurpagar
 
Are There Any Natural Remedies To Treat Syphilis.pdf
Are There Any Natural Remedies To Treat Syphilis.pdfAre There Any Natural Remedies To Treat Syphilis.pdf
Are There Any Natural Remedies To Treat Syphilis.pdf
Little Cross Family Clinic
 
Evaluation of antidepressant activity of clitoris ternatea in animals
Evaluation of antidepressant activity of clitoris ternatea in animalsEvaluation of antidepressant activity of clitoris ternatea in animals
Evaluation of antidepressant activity of clitoris ternatea in animals
Shweta
 
Superficial & Deep Fascia of the NECK.pptx
Superficial & Deep Fascia of the NECK.pptxSuperficial & Deep Fascia of the NECK.pptx
Superficial & Deep Fascia of the NECK.pptx
Dr. Rabia Inam Gandapore
 
How to Give Better Lectures: Some Tips for Doctors
How to Give Better Lectures: Some Tips for DoctorsHow to Give Better Lectures: Some Tips for Doctors
How to Give Better Lectures: Some Tips for Doctors
LanceCatedral
 
BENIGN PROSTATIC HYPERPLASIA.BPH. BPHpdf
BENIGN PROSTATIC HYPERPLASIA.BPH. BPHpdfBENIGN PROSTATIC HYPERPLASIA.BPH. BPHpdf
BENIGN PROSTATIC HYPERPLASIA.BPH. BPHpdf
DR SETH JOTHAM
 
ANATOMY AND PHYSIOLOGY OF URINARY SYSTEM.pptx
ANATOMY AND PHYSIOLOGY OF URINARY SYSTEM.pptxANATOMY AND PHYSIOLOGY OF URINARY SYSTEM.pptx
ANATOMY AND PHYSIOLOGY OF URINARY SYSTEM.pptx
Swetaba Besh
 
New Directions in Targeted Therapeutic Approaches for Older Adults With Mantl...
New Directions in Targeted Therapeutic Approaches for Older Adults With Mantl...New Directions in Targeted Therapeutic Approaches for Older Adults With Mantl...
New Directions in Targeted Therapeutic Approaches for Older Adults With Mantl...
i3 Health
 
HOT NEW PRODUCT! BIG SALES FAST SHIPPING NOW FROM CHINA!! EU KU DB BK substit...
HOT NEW PRODUCT! BIG SALES FAST SHIPPING NOW FROM CHINA!! EU KU DB BK substit...HOT NEW PRODUCT! BIG SALES FAST SHIPPING NOW FROM CHINA!! EU KU DB BK substit...
HOT NEW PRODUCT! BIG SALES FAST SHIPPING NOW FROM CHINA!! EU KU DB BK substit...
GL Anaacs
 
Phone Us ❤85270-49040❤ #ℂall #gIRLS In Surat By Surat @ℂall @Girls Hotel With...
Phone Us ❤85270-49040❤ #ℂall #gIRLS In Surat By Surat @ℂall @Girls Hotel With...Phone Us ❤85270-49040❤ #ℂall #gIRLS In Surat By Surat @ℂall @Girls Hotel With...
Phone Us ❤85270-49040❤ #ℂall #gIRLS In Surat By Surat @ℂall @Girls Hotel With...
Savita Shen $i11
 
Charaka Samhita Sutra sthana Chapter 15 Upakalpaniyaadhyaya
Charaka Samhita Sutra sthana Chapter 15 UpakalpaniyaadhyayaCharaka Samhita Sutra sthana Chapter 15 Upakalpaniyaadhyaya
Charaka Samhita Sutra sthana Chapter 15 Upakalpaniyaadhyaya
Dr KHALID B.M
 
Physiology of Chemical Sensation of smell.pdf
Physiology of Chemical Sensation of smell.pdfPhysiology of Chemical Sensation of smell.pdf
Physiology of Chemical Sensation of smell.pdf
MedicoseAcademics
 
Maxilla, Mandible & Hyoid Bone & Clinical Correlations by Dr. RIG.pptx
Maxilla, Mandible & Hyoid Bone & Clinical Correlations by Dr. RIG.pptxMaxilla, Mandible & Hyoid Bone & Clinical Correlations by Dr. RIG.pptx
Maxilla, Mandible & Hyoid Bone & Clinical Correlations by Dr. RIG.pptx
Dr. Rabia Inam Gandapore
 
Alcohol_Dr. Jeenal Mistry MD Pharmacology.pdf
Alcohol_Dr. Jeenal Mistry MD Pharmacology.pdfAlcohol_Dr. Jeenal Mistry MD Pharmacology.pdf
Alcohol_Dr. Jeenal Mistry MD Pharmacology.pdf
Dr Jeenal Mistry
 
The Normal Electrocardiogram - Part I of II
The Normal Electrocardiogram - Part I of IIThe Normal Electrocardiogram - Part I of II
The Normal Electrocardiogram - Part I of II
MedicoseAcademics
 
24 Upakrama.pptx class ppt useful in all
24 Upakrama.pptx class ppt useful in all24 Upakrama.pptx class ppt useful in all
24 Upakrama.pptx class ppt useful in all
DrSathishMS1
 
ARTHROLOGY PPT NCISM SYLLABUS AYURVEDA STUDENTS
ARTHROLOGY PPT NCISM SYLLABUS AYURVEDA STUDENTSARTHROLOGY PPT NCISM SYLLABUS AYURVEDA STUDENTS
ARTHROLOGY PPT NCISM SYLLABUS AYURVEDA STUDENTS
Dr. Vinay Pareek
 
Surat @ℂall @Girls ꧁❤8527049040❤꧂@ℂall @Girls Service Vip Top Model Safe
Surat @ℂall @Girls ꧁❤8527049040❤꧂@ℂall @Girls Service Vip Top Model SafeSurat @ℂall @Girls ꧁❤8527049040❤꧂@ℂall @Girls Service Vip Top Model Safe
Surat @ℂall @Girls ꧁❤8527049040❤꧂@ℂall @Girls Service Vip Top Model Safe
Savita Shen $i11
 

Recently uploaded (20)

Flu Vaccine Alert in Bangalore Karnataka
Flu Vaccine Alert in Bangalore KarnatakaFlu Vaccine Alert in Bangalore Karnataka
Flu Vaccine Alert in Bangalore Karnataka
 
Cervical & Brachial Plexus By Dr. RIG.pptx
Cervical & Brachial Plexus By Dr. RIG.pptxCervical & Brachial Plexus By Dr. RIG.pptx
Cervical & Brachial Plexus By Dr. RIG.pptx
 
Antiulcer drugs Advance Pharmacology .pptx
Antiulcer drugs Advance Pharmacology .pptxAntiulcer drugs Advance Pharmacology .pptx
Antiulcer drugs Advance Pharmacology .pptx
 
Are There Any Natural Remedies To Treat Syphilis.pdf
Are There Any Natural Remedies To Treat Syphilis.pdfAre There Any Natural Remedies To Treat Syphilis.pdf
Are There Any Natural Remedies To Treat Syphilis.pdf
 
Evaluation of antidepressant activity of clitoris ternatea in animals
Evaluation of antidepressant activity of clitoris ternatea in animalsEvaluation of antidepressant activity of clitoris ternatea in animals
Evaluation of antidepressant activity of clitoris ternatea in animals
 
Superficial & Deep Fascia of the NECK.pptx
Superficial & Deep Fascia of the NECK.pptxSuperficial & Deep Fascia of the NECK.pptx
Superficial & Deep Fascia of the NECK.pptx
 
How to Give Better Lectures: Some Tips for Doctors
How to Give Better Lectures: Some Tips for DoctorsHow to Give Better Lectures: Some Tips for Doctors
How to Give Better Lectures: Some Tips for Doctors
 
BENIGN PROSTATIC HYPERPLASIA.BPH. BPHpdf
BENIGN PROSTATIC HYPERPLASIA.BPH. BPHpdfBENIGN PROSTATIC HYPERPLASIA.BPH. BPHpdf
BENIGN PROSTATIC HYPERPLASIA.BPH. BPHpdf
 
ANATOMY AND PHYSIOLOGY OF URINARY SYSTEM.pptx
ANATOMY AND PHYSIOLOGY OF URINARY SYSTEM.pptxANATOMY AND PHYSIOLOGY OF URINARY SYSTEM.pptx
ANATOMY AND PHYSIOLOGY OF URINARY SYSTEM.pptx
 
New Directions in Targeted Therapeutic Approaches for Older Adults With Mantl...
New Directions in Targeted Therapeutic Approaches for Older Adults With Mantl...New Directions in Targeted Therapeutic Approaches for Older Adults With Mantl...
New Directions in Targeted Therapeutic Approaches for Older Adults With Mantl...
 
HOT NEW PRODUCT! BIG SALES FAST SHIPPING NOW FROM CHINA!! EU KU DB BK substit...
HOT NEW PRODUCT! BIG SALES FAST SHIPPING NOW FROM CHINA!! EU KU DB BK substit...HOT NEW PRODUCT! BIG SALES FAST SHIPPING NOW FROM CHINA!! EU KU DB BK substit...
HOT NEW PRODUCT! BIG SALES FAST SHIPPING NOW FROM CHINA!! EU KU DB BK substit...
 
Phone Us ❤85270-49040❤ #ℂall #gIRLS In Surat By Surat @ℂall @Girls Hotel With...
Phone Us ❤85270-49040❤ #ℂall #gIRLS In Surat By Surat @ℂall @Girls Hotel With...Phone Us ❤85270-49040❤ #ℂall #gIRLS In Surat By Surat @ℂall @Girls Hotel With...
Phone Us ❤85270-49040❤ #ℂall #gIRLS In Surat By Surat @ℂall @Girls Hotel With...
 
Charaka Samhita Sutra sthana Chapter 15 Upakalpaniyaadhyaya
Charaka Samhita Sutra sthana Chapter 15 UpakalpaniyaadhyayaCharaka Samhita Sutra sthana Chapter 15 Upakalpaniyaadhyaya
Charaka Samhita Sutra sthana Chapter 15 Upakalpaniyaadhyaya
 
Physiology of Chemical Sensation of smell.pdf
Physiology of Chemical Sensation of smell.pdfPhysiology of Chemical Sensation of smell.pdf
Physiology of Chemical Sensation of smell.pdf
 
Maxilla, Mandible & Hyoid Bone & Clinical Correlations by Dr. RIG.pptx
Maxilla, Mandible & Hyoid Bone & Clinical Correlations by Dr. RIG.pptxMaxilla, Mandible & Hyoid Bone & Clinical Correlations by Dr. RIG.pptx
Maxilla, Mandible & Hyoid Bone & Clinical Correlations by Dr. RIG.pptx
 
Alcohol_Dr. Jeenal Mistry MD Pharmacology.pdf
Alcohol_Dr. Jeenal Mistry MD Pharmacology.pdfAlcohol_Dr. Jeenal Mistry MD Pharmacology.pdf
Alcohol_Dr. Jeenal Mistry MD Pharmacology.pdf
 
The Normal Electrocardiogram - Part I of II
The Normal Electrocardiogram - Part I of IIThe Normal Electrocardiogram - Part I of II
The Normal Electrocardiogram - Part I of II
 
24 Upakrama.pptx class ppt useful in all
24 Upakrama.pptx class ppt useful in all24 Upakrama.pptx class ppt useful in all
24 Upakrama.pptx class ppt useful in all
 
ARTHROLOGY PPT NCISM SYLLABUS AYURVEDA STUDENTS
ARTHROLOGY PPT NCISM SYLLABUS AYURVEDA STUDENTSARTHROLOGY PPT NCISM SYLLABUS AYURVEDA STUDENTS
ARTHROLOGY PPT NCISM SYLLABUS AYURVEDA STUDENTS
 
Surat @ℂall @Girls ꧁❤8527049040❤꧂@ℂall @Girls Service Vip Top Model Safe
Surat @ℂall @Girls ꧁❤8527049040❤꧂@ℂall @Girls Service Vip Top Model SafeSurat @ℂall @Girls ꧁❤8527049040❤꧂@ℂall @Girls Service Vip Top Model Safe
Surat @ℂall @Girls ꧁❤8527049040❤꧂@ℂall @Girls Service Vip Top Model Safe
 

Test bank for focus on nursing pharmacology 7th edition by amy m karch.pdf

  • 1. Test Bank - Karch Focus on Nursing Pharmacology 9th Edition by Rebecca Tucker Chapters 1 - 59
  • 2. Table of Contents Chapter 01- Introduction to Drugs Chapter 02- Drugs and the Body Chapter 03- Toxic Effects of Drugs Chapter 04- The Nursing Process in Drug Therapy and Patient Safety Chapter 05- Dosage Calculations Chapter 06- Challenges to Effective Drug Therapy Chapter 07- Introduction to Cell Physiology Chapter 08- Anti-infective Agents Chapter 09- Antibiotics Chapter 10- Antiviral Agents Chapter 11- Antifungal Agents Chapter 12- Antiprotozoal Agents Chapter 13- Anthelmintic Agents Chapter 14- Antineoplastic Agents Chapter 15- Introduction to the Immune Response and Inflammation Chapter 16- Anti-inflammatory, Antiarthritis, and Related Agents Chapter 17- Immune Modulators Chapter 18- Vaccines and Sera Chapter 19- Introduction to Nerves and the Nervous System Chapter 20- Anxiolytic and Hypnotic Agents Chapter 21- Antidepressant Agents Chapter 22- Psychotherapeutic Agents Chapter 23- Antiseizure Agents Chapter 24- Antiparkinsonism Agents Chapter 25- Muscle Relaxants Chapter 26- Narcotics, Narcotic Antagonists, and Antimigraine Agents Chapter 27- General and Local Anesthetic Agents Chapter 28- Neuromuscular Junction Blocking Agents Chapter 29- Introduction to the Autonomic Nervous System Chapter 30- Adrenergic Agonists Chapter 31- Adrenergic Antagonists Chapter 32- Cholinergic Agonists Chapter 33- Anticholinergic Agents Chapter 34- Introduction to the Endocrine System Chapter 35- Hypothalamic and Pituitary Agents Chapter 36- Adrenocortical Agents Chapter 37- Thyroid and Parathyroid Agents Chapter 38- Agents to Control Blood Glucose Levels Chapter 39- Introduction to the Reproductive System Chapter 40- Drugs Affecting the Female Reproductive System Chapter 41- Drugs Affecting the Male Reproductive System Chapter 42- Introduction to the Cardiovascular System Chapter 43- Drugs Affecting Blood Pressure Chapter 44- Agents for Treating Heart Failure Chapter 45- Antiarrhythmic Agents Chapter 46- Antianginal Agents Chapter 47- Lipid-Lowering Agents Chapter 48- Drugs Affecting Blood Coagulation Chapter 49- Drugs Used to Treat Anemias Chapter 50- Introduction to the Renal System Chapter 51- Diuretic Agents Chapter 52- Drugs Affecting the Urinary Tract and the Bladder Chapter 53- Introduction to the Respiratory System Chapter 54- Drugs Acting on the Upper Respiratory Tract Chapter 55- Drugs Acting on the Lower Respiratory Tract Chapter 56- Introduction to the Gastrointestinal System Chapter 57- Drugs Affecting Gastrointestinal Secretions Chapter 58- Drugs Affecting Gastrointestinal Motility Chapter 59- Antiemetic Agents
  • 3. Chapter 1: Introduction to Drugs Cognitive Level: Application Difficulty: Moderate Integrated Process: Nursing process 1.A nurse working in radiology administers iodine to a patient who is having a computerized axial tomography (CAT) scan. A nurse working on an oncology unit administers chemotherapy to patients who have cancer. At the Public Health Department, a nurse administers a measles-mumps-rubella (MMR) vaccine to a 14- month-old child as a routine immunization. Which branch of pharmacology best describes the actions of all three nurses? A) Pharmacoeconomics B) Pharmacotherapeutics C) Pharmacodynamics D) Pharmacokinetics ANSWER: B Response: Pharmacology is the study of the biologic effects of chemicals. Nurses are involved with clinical pharmacology or pharmacotherapeutics, which is a branch of pharmacology that deals with the uses of drugs to treat, prevent, and diagnose disease. The nurse working in radiology is administering a drug to help diagnose a disease. The nurse working on an oncology unit is administering a drug to help treat a disease. Pharmacoeconomics includes any costs involved in drug therapy. Pharmacodynamics involves how a drug affects the body, and pharmacokinetics is how the body acts on the drug. Cognitive Level: Comprehension Difficulty: Easy Integrated Process: Nursing process 2.A physician has ordered intramuscular injections of morphine, a narcotic, every 4 hours as needed for pain for a motor vehicle accident victim. The nurse is aware that there is a high abuse potential for this drug; therefore, morphine is categorized as a: A) Schedule I drug B) Schedule II drug C) Schedule III drug D) Schedule IV drug
  • 4. ANSWER: B Response: Narcotics such as morphine are considered schedule II drugs because of the high abuse potential with severe dependence liability. Schedule I drugs have high abuse potential and no accepted medical use. Schedule III drugs have a lesser abuse potential than schedule II drugs and an accepted medical use. Schedule IV drugs have low abuse potential and limited dependence liability. Cognitive Level: Comprehension Difficulty: Easy Integrated Process: Communication and documentation 3.A nurse working for a drug company is involved in phase III drug evaluation studies. Which of the following might the nurse be responsible for during this stage of drug development?
  • 5. Chapter 1: Introduction to Drugs A) Working with animals who are given experimental drugs B) Monitoring drug effects in patients who are selected to participate in a study, who have the disease that the drug is meant to treat C) Collecting records of symptoms that participants experience while taking a drug and determining whether they are caused by the disease or the drug D) Informing healthy, young volunteer participants of possible risks that could occur from taking an experimental drug ANSWER: C Response: Phase III studies involve use of a drug in a vast clinical market where patients are asked to record any symptoms they experience while taking the drugs. Nurses may be responsible for helping collect and analyze the information to be shared with the Food and Drug Administration (FDA). Use of animals in drug testing is done in the preclinical trials. A select group of patients who are involved in phase II studies participate in studies where the participants have the disease the drug is intended to treat. These patients are monitored closely for drug effects. Phase I studies involve healthy human volunteers who are usually paid for their participation. Nurses may observe for adverse effects and toxicity. Cognitive Level: Comprehension Difficulty: Moderate Integrated Process: Nursing Process 4.Which of the following concepts is considered when generic drugs are substituted for brand-name drugs? A) Bioavailability B) Critical concentration C) Distribution D) Half-life ANSWER: A Response: Bioavailability is the portion of a dose of a drug that reaches the systemic circulation and is available to act on body cells. Binders used in a generic drug may not be the same as those used in the brand-name drug. Therefore, the way the body breaks down and uses the drug may differ, which may eliminate a substitution. Critical
  • 6. concentration is the amount of a drug that is needed to cause a therapeutic effect. Distribution is the phase of pharmacokinetics that involves the movement of a drug to the body's tissues. A drug's half-life is the time it takes for the amount of drug to decrease to one-half of the peak level. Cognitive Level: Analysis Difficulty: Difficult Integrated Process: Teaching/learning 5.A nurse is teaching her patient about the use of over-the-counter (OTC) drugs. Which of the following statements best informs the patient about their safe use? A) “OTC drugs are products that are available without prescription for self- treatment of minor complaints.” B) “OTC drugs are considered medications and should be reported on a drug history.”
  • 7. Chapter 1: Introduction to Drugs C) “OTC drugs were approved as prescription drugs but later were found to be safe without the need for a prescription.” D) “OTC drugs need to be taken with caution. They can mask the signs and symptoms of an underlying disease and interfere with prescription drug therapy.” ANSWER: D Response: OTC drugs are considered medications and should be reported. OTC drugs are available without a prescription, although some were first approved as prescription drugs. The most important teaching should relate to their safe use and that OTC drugs can mask symptoms of disease and interfere with prescribed drugs. Cognitive Level: Knowledge Difficulty: Easy Integrated Process: Nursing Process 6.Which of the following legislative acts allowed the Food and Drug Administration (FDA) to tighten control over the quality of drugs and required that safety and efficacy standards be established? A) Pure Food and Drug Act of 1906 B) Federal Food, Drug, and Cosmetic Act of 1938 C) Durham Humphrey Amendment of 1951 D) Kefauver-Harris Act of 1962 ANSWER: D Response: The Kefauver-Harris Act was the result of the use of the 1960s drug thalidomide (Thalomid). The public concern led to the legislation that gave the FDA regulatory control over testing and evaluating of drugs and allowed it to set standards for efficacy and safety. The Pure Food and Drug Act required labeling to eliminate false claims. The Federal Food, Drug, and Cosmetic Act gave the FDA the power to enforce standards for testing drug toxicity and monitoring labeling. The Durham- HumphreyAmendment enforced prescriptions for distribution. Cognitive Level: Application Difficulty: Moderate
  • 8. Integrated Process: Teaching/learning 7.A nurse is instructing a pregnant patient concerning the potential risk to her fetus from a pregnancy category B drug. The nurse would inform the patient that: A) “Adequate studies in pregnant women have demonstrated there is no risk to the fetus.” B) “Animal studies have not demonstrated a risk to the fetus but there have been no adequate studies in pregnant women.” C) “Animal studies have shown an adverse effect on the fetus but there are no adequate studies in pregnant women.” D) “There is evidence of human fetal risk but the potential benefits from use of the drug may be acceptable despite potential risks.” ANSWER: B Response: Category B indicates that animal studies have not demonstrated a risk to the fetus but
  • 9. Chapter 1: Introduction to Drugs there have been no adequate studies in pregnant women. However, there have not been adequate studies in pregnant women to demonstrate risk to a fetus during the first trimester of pregnancy and no evidence of risk in later trimesters. Category A indicates that adequate studies in pregnant women have not demonstrated a risk to the fetus in the first trimester or in later trimesters. Category C indicates that animal studies have shown an adverse effect on the fetus but there have been no adequate studies in humans. Category D reveals evidence of human fetal risk, but the potential benefits from the use of the drugs in pregnant women may outweigh the risks. Cognitive Level: Analysis Difficulty: Difficult Integrated Process: Teaching/learning 8. Discharge planning for patients leaving the hospital should include instructions on the use of over-the-counter (OTC) drugs. Which comment by the patient would demonstrate a good understanding of OTC drugs? A) “OTC drugs are always safe and will not cause bad effects.” B) “OTC drugs have been around for years and have not been tested by the Food and Drug Administration (FDA).” C) “OTC drugs are different from any drugs available by prescription and cost less.” D) “OTC drugs are thought to be safe when taken as directed.” ANSWER: D Response: OTC drugs are drugs that have been determined to be safe when taken as directed. They may have originally been prescription drugs that were tested by the FDA or they may have been grandfathered in when the FDA laws changed. OTC education should always be included as a part of the hospital discharge instructions. Cognitive Level: Comprehension Difficulty: Moderate Integrated Process: Teaching/Learning 9. Which of the following would be the best source of drug information for a nurse? A) Drug Facts and Comparisons book B) A nurse's drug guide
  • 10. C) A drug package insert D) The Physician's Drug Reference (PDR) ANSWER: B Response: The most user-friendly drug source for a nurse is a nurse's drug guidebook. A guide provides nursing implications and patient teaching points that are most useful to nurses. The Drug Facts and Comparisons book provides a wide range of drug information but is hard to manipulate and is very expensive. A package insert contains all of the chemical and drug company research information about a drug; however, the information can be difficult to understand and the print is very small. The PDR is heavily cross-referenced and difficult to use. Cognitive Level: Comprehension
  • 11. Chapter 1: Introduction to Drugs Difficulty: Moderate Integrated Process: Nursing Process 10. Which of the following statements best defines how a chemical becomes a drug? A) A chemical must have a proven therapeutic value or efficacy without severe toxicity or damaging properties to become a drug. B) A chemical becomes a drug when it is introduced into the body to cause a change. C) A chemical is considered a drug when the Food and Drug Administration (FDA) approves its release to be marketed. D) A chemical must have demonstrated therapeutic value to become a drug. ANSWER: A Response: A chemical must undergo a series of tests to determine its therapeutic value and efficacy without severe toxicity or damaging properties before it is termed a drug. Test results are reported to the FDA, which may or may not give approval.
  • 12. Chapter 2: Drugs and the Body Cognitive Level: Analysis Difficulty: Difficult Integrated Process: Nursing process 1. In which of the following patients would a nurse expect to see an alteration in drug metabolism? A) A 35-year-old female with cervical cancer B) A 41-year-old male with kidney stones C) A 50-year-old male with cirrhosis of the liver D) A 62-year-old female in acute renal failure ANSWER: C Response: The liver is the most important site of drug metabolism. If the liver is not functioning effectively, as in patients with cirrhosis, drugs will not be metabolized normally and toxic levels could develop. In cirrhosis patients the drug dosage will need to be altered to ensure normal levels in the body. Cognitive Level: Analysis Difficulty: Difficult Integrated Process: Nursing process 2.A patient has a drug level of 50 units/mL and the drug's half-life is 1 hour. If concentrations above 25 units/mL are considered toxic and no more drug is given, how long will it take for the blood level to reach the nontoxic range? A) 30 minutes B) 1 hour C) 2 hours D) 3 hours ANSWER: C Response: Half-life is the time required for the serum concentration of a drug to decrease by 50%. After 1 hour, the serum concentration would be 25 units/mL (50/2). After 2 hours, the serum concentration would be 12.5 units/mL (25/2) and reach the nontoxic range.
  • 13. Cognitive Level: Analysis Difficulty: Difficult Integrated Process: Nursing process 3.A nurse is caring for a patient who has recently moved from Vermont to south Florida. The patient has been on the same antihypertensive drug for 6 years and has had stable blood pressures and no side effects. Since her move, however, she reports having “dizzy spells and weakness” and feels that the drug is no longer effective. The nurse knows that one possible reason for the change in the effectiveness of the drug could be which of the following? A) The impact of the placebo effect on the patient's response B) The accumulative effect of the drug if it has been taken for many years C) The impact of the warmer environment on the patient's physical state D) Problems with patient compliance with the drug regimen while on vacation ANSWER: C Response:
  • 14. Chapter 2: Drugs and the Body Antihypertensive drugs work to decrease the blood pressure. When a patient goes to a climate that is much warmer than usual, blood vessels dilate and the blood pressure falls. If a patient is taking an antihypertensive drug and enters a warmer climate, there is a chance that the patient's blood pressure will drop too low, resulting in dizziness and feelings of weakness. Most antihypertensives are metabolized and excreted and do not accumulate in the body. Patients must be very compliant with their drug regimen while on vacation. After several years on an antihypertensive drug, the effects of that drug are known; therefore, the placebo effect should not be an issue. Cognitive Level: Comprehension Difficulty: Moderate Integrated Process: Nursing Process 4. During which phase of pharmacokinetics could a significant drug-drug interaction occur, resulting in one drug stimulating or blocking the metabolism of the other drug? A) Absorption B) Distribution C) Biotransformation D) Excretion ANSWER: C Response: During biotransformation, a drug may be metabolized quicker if taken at the same time as another drug, and higher doses of the drug will be needed to achieve the desired effect. During absorption, one drug can prevent or accelerate absorption of the other drug. During distribution, one drug competes for the protein-binding site of another drug so the second drug cannot be transported to the reactive tissue. During excretion, one drug competes for excretion with the other drug, leading to accumulation and toxic effects of one of the drugs. Cognitive Level: Comprehension Difficulty: Moderate Integrated Process: Nursing Process 5. Which statement accurately describes a drug agonist? A) A drug that reacts with specific receptor sites on a cell and prevents the
  • 15. reaction of another chemical with a different receptor site on that cell B) A drug that interferes with the enzyme systems that act as catalysts for different chemical reactions C) A drug that interacts directly with receptor sites to cause the same activity that a natural chemical would cause at that site D) A drug that reacts with receptor sites to block normal stimulation, producing no effect ANSWER: C Response: Agonists are drugs that produce effects similar to those produced by naturally occurring neurotransmitters, hormones, or other substances found in the body. Noncompetitive antagonists are drugs that react with certain receptor sites, preventing the reaction of another chemical with a different receptor site. Drug- enzyme interactions interfere with the enzyme systems that stimulate various
  • 16. Chapter 2: Drugs and the Body chemical reactions. Cognitive Level: Analysis Difficulty: Difficult Integrated Process: Nursing process 6.A nurse has been administering a drug to a patient intramuscularly (IM). The physician discontinued the IM dose and wrote an order for the drug to be given orally. The nurse notices that the oral dosage is considerably higher than the parenteral dose and understands that this is due to: A) Passive diffusion B) Active transport C) Glomerular filtration D) First-pass effect ANSWER: D Response: The first-pass effect involves drugs that are absorbed from the small intestine directly into the portal venous system, which delivers the drug molecules to the liver. Once in the liver, enzymes break the drug into metabolites, which may become active or may be deactivated and readily excreted from the body. A large percentage of the oral dose is usually destroyed and never reaches tissues. Oral dosages account for the phenomenon to ensure an appropriate amount of the drug in the body to produce a therapeutic action. Passive diffusion is the major process through which drugs are absorbed into the body. Active transport is a process that uses energy to actively move a molecule across a cell membrane and is often involved in drug excretion in the kidney. Glomerular filtration is the passage of water and water-soluble components from the plasma into the renal tubule. Cognitive Level: Comprehension Difficulty: Easy Integrated Process: Nursing process 7.A nurse working as a member of a research team is exploring the unique differences in drug response that each individual possesses based on his or her genetic makeup. This area of study is called: A) Pharmacotherapeutics
  • 17. B) Pharmacodynamics C) Pharmacoeconomics D) Pharmacogenomics ANSWER: D Response: Pharmacogenomics is the area of study that includes mapping of the human genome. In the future, medical care and drug regimens may be personally designed based on a person's unique genetic makeup. Pharmacotherapeutics is the branch of pharmacology that deals with the use of drugs to treat, prevent, and diagnose disease. Pharmacodynamics involves how a drug affects the body. Pharmacoeconomics includes any costs involved in drug therapy. Cognitive Level: Comprehension
  • 18. Chapter 2: Drugs and the Body Difficulty: Easy Integrated Process: Nursing process 8.The nurse is aware that for a drug to work properly on reactive tissues, a sufficient amount of the drug is needed to cause the therapeutic effect. The term used for this amount is: A) Critical concentration B) Dynamic equilibrium C) Selective toxicity D) Active transport ANSWER: A Response: A critical concentration of a drug must be present before a reaction occurs within our cells. A dynamic equilibrium is obtained from absorption of a drug from the site of drug entry, distribution to the active site, metabolism in the liver, and excretion from the body to have a critical concentration. Selective toxicity is the ability of a drug to attach only to those systems found in foreign cells. Active transport is the process that uses energy to actively move a molecule across a cell membrane and is often involved in drug excretion in the kidney. Cognitive Level: Analysis Difficulty: Difficult Integrated Process: Nursing process 9.A nurse has received an order to administer two drugs at the same time of the day. The nurse should first: A) Wash her hands before handling the medications B) Consult a drug guide for compatibility C) Question the patient concerning drug allergies D) Identify the patient by checking the armband and asking the patient to state his name ANSWER: B Response: A nurse should always first consult a drug guide for compatibility when two or more drugs are being given at the same time. Once she is certain the drugs are compatible, she should wash her hands and prepare the medication. She should identify the patient appropriately and ask about drug allergies prior to administering the
  • 19. medication. Cognitive Level: Comprehension Difficulty: Moderate Integrated Process: Nursing Process 10.Which of the following factors affects absorption of intramuscular (IM) administration of drugs? A) Perfusion of blood to the subcutaneous tissue B) Integrity of the mucous membranes C) Environmental temperature D) Blood flow to the gastrointestinal tract ANSWER: C Response:
  • 20. Chapter 2: Drugs and the Body A cold environmental temperature can cause muscles to vasoconstrict and decreases absorption; a hot environment can cause vasodilation and increase absorption of IM medications. Blood flow to the subcutaneous tissues interferes with subcutaneous injection, and blood flow to the gastrointestinal tract causes alterations in absorption for oral medications. The condition of mucous membranes can interfere with sublingual (under the tongue) and buccal (in the cheek) administration of drugs.
  • 21. Chapter 3: Toxic Effects of Drugs Cognitive Level: Application Difficulty: Difficult Integrated Process: Teaching/learning 1.A nurse is instructing her patient about a newly prescribed drug. Which of the following should be included in the teaching to help improve patient compliance and safety? A) List of pharmacies where the drug can be obtained B) Measures to alleviate any discomfort associated with adverse effects C) The cost of the brand-name drug compared to the generic form D) Statistics related to phase III of testing for the prescribed drug ANSWER: B Response: If patients are aware of certain adverse effects and how to alleviate or decrease the discomfort, they are more likely to continue taking the medication. A list of pharmacies can be useful information but will not improve safety or compliance. Knowing the cost of the brand-name versus the generic drug could also be helpful to the patient; however, a substitution may not be allowable and the cost of a drug does not improve patient safety. Most patients are not concerned with the statistics related to drug testing and the results are therefore not shared with the patient. Cognitive Level: Comprehension Difficulty: Moderate Integrated Process: Nursing process 2.A patient has been taking an oral antibiotic for 10 days. She reports to the nurse that she has developed vaginal itching and a clear discharge. The nurse is aware that the patient is experiencing which of the following? A) An adverse reaction from the antibiotic B) A stomatitis, which has caused an inflammation of the mucous membranes C) An overdose of the drug that is damaging to more than one body system D) A superinfection caused by the antibiotic, which has destroyed normal flora ANSWER: D Response: Superinfections often occur with antibiotic use because the drug kills normal flora bacteria. Vaginal itching and a clear discharge are signs and symptoms of a yeast
  • 22. infection and not stomatitis. Vaginal itching and a clear discharge are not considered adverse effects of an antibiotic. An overdose of a drug that damages more than one body system is considered poisoning. Cognitive Level: Analysis Difficulty: Moderate Integrated Process: Nursing process 3.A male patient is admitted to the emergency room in distress. The nurse suspects an anaphylactic reaction from oral penicillin. Which assessment findings are important in making this diagnosis? A) Blood pressure: 186/100, difficulty breathing B) Hematocrit (Hct): 32%, decreased urine output C) Temperature: 102ºF, swollen joints D) Profuse sweating, blood pressure: 92/58
  • 23. Chapter 3: Toxic Effects of Drugs ANSWER: A Response: An anaphylactic reaction is an immune reaction that causes a massive release of histamine, resulting in edema and swelling, which can lead to respiratory distress and increased blood pressure. A decreased Hct and decreased urine output suggest a cytotoxic reaction. An increased temperature and swollen joints could suggest serum sickness. Profuse sweating and a blood pressure of 92/58 may indicate cardiac- related issues Cognitive Level: Application Difficulty: Moderate Integrated Process: Teaching/learning 4.A patient is taking an antihistamine to relieve itchy, watery eyes and a runny nose associated with seasonal allergies. A good nursing intervention would be to advise the patient to: A) Limit fluid intake to dry mucous membranes B) Avoid driving or operating machinery C) Report strange dreams or nightmares D) Decrease dietary fat ANSWER: B Response: An adverse effect of antihistamines is drowsiness, and injury can occur if the patient drives or operates machinery. An increase in fluids would be indicated to help keep nasal membranes moist. It is common for dreams to occur when taking medication and it is not necessary to report them. Dietary fat should not interfere with the drug metabolism of antihistamines. Cognitive Level: Application Difficulty: Difficult Integrated Process: Teaching/learning 5.A nurse is teaching a group of five patients who are beginning drug therapy for AIDS. Which of the following should be included in her instructions to the group? A) “Take your medications as indicated. Poisoning can occur when an overdose damages more than one body system.”
  • 24. B) “Renal injury is common and can be caused by the first-pass effect.” C) “A blood dyscrasia due to drug therapy can be serious. Call us if your skin looks yellow.” D) “Most drugs are metabolized in the liver. You should report dark red papules immediately.” ANSWER: A Response: Poisoning can lead to the potential for fatal reactions when more than one body system is affected. Liver injury can be caused by the first-pass effect and can cause the skin to have a yellow appearance. Most drugs are metabolized in the liver. However, dark red papules appearing on extremities are characteristic of Stevens- Johnson syndrome, a potentially fatal erythema multiforme exudativum.
  • 25. Chapter 3: Toxic Effects of Drugs Cognitive Level: Analysis Difficulty: Difficult Integrated Process: Nursing process 6.Potassium-sparing diuretics can lead to potassium retention. Which assessment finding could indicate hyperkalemia? A) Urine output of 1500 mL/24 hours B) Blood pressure of 98/60 C) Potassium level of 5.9 mEq/L D) Pulse: 60 and weak ANSWER: C Response: The normal range of serum potassium for an adult is 3.5 to 5.0 mEq/L. A level higher than 5.0 mEq/L can indicate hyperkalemia. Normal urinary output is between 1500 and 2000 mL per day. Urinary output below 1000 mL per day would include oliguria and would indicate hyperkalemia. A decrease in blood pressure and pulse can indicate hypokalemia. Cognitive Level: Application Difficulty: Moderate Integrated Process: Teaching/learning 7.A nurse is talking with family members of an 80-year-old patient who is experiencing tinnitus from taking a macrolide antibiotic. Which of the following should the nurse include in her instructions regarding home care? A) Keep the patient in the prone position when in bed. B) Eliminate salt from the patient's diet. C) Provide protective measures to prevent falling or injury. D) Monitor exposure to sunlight. ANSWER: C Response: Macrolide antibiotics can cause severe auditory nerve damage, which can cause dizziness, ringing in the ears (tinnitus), and loss of balance and hearing. The patient would be at high risk for injury due to falls. Usually a person who is dizzy is unable to lie flat and needs to recline with the head elevated. Salt may be decreased but not entirely eliminated from the diet. Sunlight is usually not an issue because clients who experience tinnitus are inside their homes in a quiet environment or on bed rest.
  • 26. Cognitive Level: Application Difficulty: Moderate Integrated Process: Nursing process 8.Alterations in glucose metabolism can occur from use of oral antidiabetic drugs. A nurse will observe patients who are taking these drugs for which of the following? A) Increased urination B) Deep Kussmaul's respirations C) Increased hunger D) Tremors and lack of coordination ANSWER: D Response: Lowering the blood glucose is an immediate problem for the nervous system, and
  • 27. Chapter 3: Toxic Effects of Drugs tremors and lack of coordination may be observed in the patient. When high blood glucose levels occur, the patient may report increased hunger, urination, and thirst. Prolonged high blood glucose levels may result in Kussmaul's respirations as the patient tries to expel the excess CO2. Cognitive Level: Analysis Difficulty: Difficult Integrated Process: Nursing process 9.A patient is being seen in the clinic for rheumatoid arthritis. Which of the following statements would indicate that she may be developing a complication from her rheumatoid medication, chloroquine (Aralen)? A) “I have to urinate all the time.” B) “Sometimes I have blurred vision.” C) “I have tingling and numbness in my arms and legs.” D) “Sometimes I feel like I am losing my balance when I walk.” ANSWER: B Response: Chloroquine (Aralen) can cause ocular toxicity with blurring of vision, color vision changes, corneal damage, and blindness. Increased urination and tingling and numbness are signs of hyperkalemia and hypokalemia. Loss of balance can be caused by auditory damage due to drug toxicity. Cognitive Level: Analysis Difficulty: Difficult Integrated Process: Nursing process 10.A 68-year-old patient who must take antihistamines for severe allergies is planning a vacation to Mexico. The nurse will encourage the patient to: A) Avoid sightseeing during the hottest part of the day B) Discontinue the antihistamines if he becomes extremely restless C) Decrease the dosage of the drugs if he experiences excessive thirst D) Continue taking the antihistamines even if he begins to hallucinate ANSWER: A Response: Antihistamines can cause anticholinergic effects, which block the parasympathetic
  • 28. nervous system by blocking cholinergic receptors, and decrease sweating, placing the patient at high risk for heat stroke. Avoiding the hottest part of the day will help prevent dehydration and heat prostration. Extreme restlessness could indicate Parkinson-like syndrome not usually associated with antihistamines. Excessive thirst is characteristic of hyperglycemia, and hallucinations are associated with drugs that affect neurologic functioning. Nurses should never tell patients to decrease or discontinue a drug unless the prescriber has instructed them to do so.
  • 29. 1. A 70-year-old patient has just received a drug, which can cause sedation. What would be the priority nursing diagnosis for this patient? A) Noncompliance: Cost of the drug B) Deficient knowledge: Unfamiliar with drug therapy C) Risk for injury: Related to adverse effects of the drug D) Ineffective health maintenance: Need for medication 2. When providing drug therapy to a patient what is a responsibility of the nurse? A) Teaching the patient how to cope with the effects of the drug to ensure the best outcome B) Helping the patient analyze the physiological and pathological effects of drugs C) Warning the patient how most patients respond to the drug therapy D) Encouraging the patient to increase or decrease dosages 3. You are gathering assessment data from a medication history of a 38-year-old male with four children. What assessment information would be most important in providing care for this patient? A) The medication history of the patient's mother and/or father B) The name of the patient's pharmacy C) Insurance, financial support, and stability for the patient and his family D) The last time the patient was hospitalized 4. During what phase of the nursing process would the nurse be required to consider the efficacy of nursing interventions related to drug therapy? A) Assessment B) Nursing diagnosis C) Interventions D) Evaluation 5. Drug studies generally base the recommended adult dose of a drug needed to reach a critical concentration on what measure?
  • 30. A) A 60-year-old man B) An average-sized woman C) A 150-pound adult male D) An obese adult
  • 31. 6. A nurse is caring for a child on the pediatric unit. A drug is ordered for the child, but there is no pediatric dose listed for the drug. To make sure that the right dose has been ordered, what will the nurse use to calculate the correct dose? A) Surface area B) Height C) Birth date D) Age at gestation 7. You are evaluating the discharge teaching you have done with your patient concerning their drug therapy. What statement from the patient would indicate that teaching had been effective? A) “I have to take three pills each day. It doesn't matter when.” B) “I should take the white pill in the morning and the green pill in the afternoon. I don't know what they are called.” C) “I need to tell all my other doctors that I am taking this Coumadin because it could interact with other drugs.” D) “I think I could cut that pill in half if I wanted to.” 8. The pharmacology students are learning about antibiotics that are known to cause photosensitivity in some patients. What nursing measures would ensure the comfort of patients taking these antibiotics and hopefully avoid serious adverse effects? A) Avoiding the use of aspirin and other over-the counter pain products B) Getting a wig or other head cover to avoid heat loss C) Avoiding exposure to sunlight or use of a sunscreen and protective clothing if exposure is inevitable D) Providing frequent rest periods in a dark room 9. When taking a medication history on a patient why should the nurse ask about the use of alternative therapies? A) Patients who use new drugs are usually not compliant with medical regimens. B) Many drug-alternative therapy interactions can cause serious problems and should be avoided. C) Natural products may be more effective and the drug may not be needed.
  • 32. D) The cost of the drug and the alternative therapy may be too great for the patient to handle.
  • 33. 10. Medication orders are written with very specific criteria and coincide with the 7 rights of medication administration. What critical piece of information is missing from the following medication order? Amoxicillin 250 mg every 8 hours. A) Dose B) Route C) Frequency of administration D) Trade name of the drug 11. You are the home health nurse caring for a 77-year-old patient with congestive heart failure. When assessing this patient, in relation to their drug therapy, what is one thing that would be important for you to include in your assessment? A) Patient's age B) Required lifestyle changes C) Family members in the community D) Compliance with nutritional recommendations 12. The student nurses are learning to weigh patients and do vital signs. Weighing a client is a nursing intervention that is most important for what? A) Dosage calculation B) Assessing changes in fluid balance C) Assessing changes in nutritional status D) Caloric needs 13. Teaching the patient/caregiver about their medications is an important step in reducing the risk of medication errors. What is an important teaching point about medications? A) Speak up B) Store medications in a warm humid place C) Adjust your medication according to your weight D) Keep a list of your prescribed medications 14. A 73-year-old male patient is being discharged home today. The discharge order for reads: Take 10 mL of guaifenesin (Robitussin) PO q4h. The Robitussin comes in bottles
  • 34. with plastic measuring caps. How much should the nurse teach the client to take at home? A) ½ tsp B) 1 tsp C) 2 tsp D) 1 tbsp
  • 35. 15. It is important to evaluate the efficacy of what parameter when evaluating the drug therapy of a patient? A) Appropriateness of drug dosages B) Compliance C) Caregivers knowledge level D) Nursing interventions 16. You are doing an admission assessment on a patient. During the medication history portion of the assessment what would be important to assess with herbal supplements? A) Research them for interactions with other medications B) Discontinue them if taking prescription medications C) Take them one hour before prescription medications D) Take them three hours after prescription medications 17. A 27-year-old male is admitted to the Emergency Department (ED) after a serious motorcycle accident. The patient has a head injury, abrasions covering the left side of his body, a broken left femur, and internal injuries that are not fully assessed at this time. He is transferred from the ED to the ICU. The nurse who is going to care for this patient in the ICU knows that one of their most important responsibilities in regard to drug therapy is what? A) Support vital functions B) Continue curative treatment C) Institute life-saving treatment D) Monitor the patient's response 18. When assessing a patient prior to starting a drug regimen it is often necessary to have a baseline kidney function study done. What might kidney impairment indicate in relation to drug therapy? A) Pharmacodynamics stable at the therapeutic dose B) Contraindication to the use of a drug C) Increased dosage of a drug D) Decreased adverse effects of a drug
  • 36. 19. A nurse is caring for a 77-year-old patient. The nurse would know that a normal physiologic change that must be considered when planning drug therapy, and is associated with aging is what? A) Blood volume decreases B) Subcutaneous tissue increases C) Total body water increases D) Muscle mass increases
  • 37. 20. You are caring for a patient who takes several drugs. You know that those patients most likely to have adverse drug reactions are who? A) Patients with coronary artery disease B) Non-compliant patients C) Patients who are on the recommended dose D) Very young and very old patients 21. Which statement best describes drug efficacy/toxicity in pediatric clients? A) Drug requirements for infants have been extensively studied. B) Drug dosage is altered by age and weight in children. C) Children always need smaller doses of medication than adults. D) Infants and children are always at greater risk for drug toxicity with any medication. 22. A 7-year-old male fell off a wood pile while playing. He has been admitted to the ICU with multiple broken bones and internal bleeding. What should the nurse know about drug therapy in this type of patient? A) Adverse effects may be decreased B) Therapeutic effects may be increased C) Pharmacodynamics may be altered D) Pharmacokinetics remain the same 23. You are designing a drug regimen for a patient. What should be considered? A) Quality-of-life issues B) Ethnicity of patient C) Gender D) Quantity-of-life issues 24. The nursing instructor is discussing drug therapy in the older adult. What would the instructor tell the students could effect therapeutic dosing in an older adult? A) In older adults, drugs have decreased GI absorption. B) In older adults, drugs are released more quickly into circulation.
  • 38. C) In older adults, drugs are distributed to a smaller portion of the tissue. D) In older adults, drugs have an increased action.
  • 39. 25. In today's healthcare environment there is often more contact between the patient and the nurse than between the patient and the physician. What does this make the nurse in the best position to do? A) Assess the patient's inability to document medication taken B) Assess the patient's inability to communicate with caregivers C) Assess the therapeutic success of the drug therapy D) Assess the patient's reluctance to purchase the prescribed drug 26. You are discussing your patient's drug regimen with their primary care provider. The patient has seen two specialists since seeing the primary care provider and been diagnosed with renal impairment. Why would this be important information? (Mark all that apply.) A) To keep everyone in the loop B) Help prevent medication errors C) So the physician can answer family questions D) Help prevent adverse effects E) Help promote a safe and effective medication regimen 27. It is often necessary to obtain baseline data prior to initiating many forms of drug therapy. These baseline data include what? (Mark all that apply.) A) Education level B) Allergies C) Drug use D) Number of members in family E) Father's occupation 28. A 32-year-old female is admitted to your unit with a diagnosis of hypovolemia. You are writing a care plan for this patient. You know that an appropriate nursing diagnosis to help prevent medication errors is what? A) Dysfunctional gastrointestinal motility B) Ineffective self health maintenance C) Risk for injury D) Deficient fluid volume
  • 40. 29. When using the nursing process in medication therapy, what does it ensure? A) That medications are given at the right time B) That there is efficient and effective care C) That the right dose of the drug is given to the patient D) The right drug is given to the right patient at the right time
  • 41. 30. A 35-year-old male patient is admitted to the hospital with pneumonia. He was originally being treated at home, but became worse when he quit taking his medicine. What would be an appropriate nursing diagnoses for this patient? A) Deficient knowledge: monitoring temperature B) Noncompliance C) Risk for injury related to office visits D) Non-adherence: overuse
  • 42. Answer Key 1. C 2. A 3. C 4. D 5. C 6. A 7. C 8. C 9. B 10. B 11. B 12. D 13. A 14. C 15. D 16. A 17. D 18. C 19. A 20. D 21. B 22. C 23. A 24. A 25. C 26. B, D, E 27. A, B, C 28. D 29. B 30. B
  • 43. Chapter 5: Dosage Calculations Cognitive Level: Knowledge Difficulty: Easy Integrated Process: Nursing process 1. Which of the following measuring systems used in drug preparation and delivery uses the gram as the basic unit of solid measure? A) Metric system B) Apothecary system C) Household system D) Avoirdupois system ANSWER: A Response: The metric system is the most widely used system of measurement and is based on the decimal system. The gram is the basic unit of solid measure and the liter is the basic unit of liquid measure. The apothecary system uses the grain as the basic unit of solid measure. The household system uses the pound as the basic unit of measure. The avoirdupois system uses ounces and grains but is mostly used by drug manufacturers for bulk medications. Cognitive Level: Analysis Difficulty: Difficult Integrated Process: Nursing process 2.A nurse is explaining to a young mother why she cannot give her 2-year-old child an adult dose of Tylenol. Which of the following statements would indicate that the mother needs further education? A) “There could be a time when my child may need a higher dose than normal.” B) “My baby's dose of Tylenol is based on a healthy adult male.” C) “My baby can't handle a high dose of Tylenol because her liver may be damaged.” D) “My child's dose of Tylenol should be based on her weight or age.” ANSWER: B Response: A child's dose is never based on an adult's dose. However, on rare occasions a child's dose might be higher than normal if a critical concentration cannot be reached with a smaller dose and a higher dose would not be harmful. Benefits from the increased
  • 44. dosage would have to outweigh the risk for adverse or toxic effects. A child's organs may not be mature enough to handle drugs, causing drug metabolism to be altered. A child's dosages are determined by the age, weight, or body surface. Cognitive Level: Comprehension Difficulty: Easy Integrated Process: Nursing process 3.A nurse is calculating a child's medication dose by using Clark's Rule. Which of the following calculations represents the application of Clark's Rule? A) Infant's age in months/150 months × the average adult dose B) Child's age in years/child's age in years + 12 times the average adult dose C) Weight of child in pounds/150 pounds × the average adult dose D) Surface area in square meters/1.73 × the average adult dose ANSWER: C
  • 45. Chapter 5: Dosage Calculations Response: Clark's Rule uses the child's weight to calculate the dose and assumes the adult dose is based on a 150-pound person. Fried's Rule applies to a child younger than 1 year of age and assumes than an adult dose would be appropriate for a child who is 12.5 years (150 months) old. Young's Rule applies to children 1 to 12 years of age. Surface area calculation of a child's dose is determined with the use of a nomogram including the child's height and weight. Cognitive Level: Application Difficulty: Moderate Integrated Process: Nursing process 4.A patient is to receive a daily dose of 240 mg of a medication. The dose is supposed to be divided evenly so that the drug is given every 6 hours. How much of the drug should be given at each dose? A) 30 mg B) 40 mg C) 50 mg D) 60 mg ANSWER: D Response: Because there are 24 hours in a day, giving a drug every 6 hours would mean giving the drug four times a day. Because the total daily dose is 240 mg, dividing that dose by four would mean each dose should be 60 mg. Cognitive Level: Application Difficulty: Moderate Integrated Process: Nursing process 5.A physician order is written for a patient to receive 500 mL of normal saline IV over 8 hours. If the IV infusion set delivers 15 drops/mL, how many drops per minute should the patient receive? A) 15 drops/min B) 20 drops/min C) 32 drops/min D) 64 drops/min
  • 46. ANSWER: A Response: If a patient were to receive 500 mL in 8 hours, dividing 500 by 8 would mean that the patient would receive 62.5 mL in 1 hour, or 60 minutes. Setting up the equation, 15 drops/mL/X = 62.5 mL/60 minutes, and cross-multiplying, the answer will be 15 drops/min. Cognitive Level: Application Difficulty: Moderate Integrated Process: Nursing process 6.Thirty-two units of Humulin insulin is ordered for the patient. The insulin is supplied in a multidose vial that is labeled 100 units/mL. How much insulin would be needed to treat this patient?
  • 47. Chapter 5: Dosage Calculations A) 0.032 cc B) 0.32 cc C) 3.2 cc D) 32 cc ANSWER: B Response: A cc is equivalent to an mL. There are 100 units in each cc. Divide that amount by 32 units for the answer (0.32 cc). Cognitive Level: Application Difficulty: Difficult Integrated Process: Nursing process 7.A young adult is receiving a maintenance dose of aminophylline, 3 mg/kg PO q6h. The patient weighs 50 kg. How much should the patient receive in a 24-hour period? A) 200 mg B) 400 mg C) 600 mg D) 800 mg ANSWER: C Response: Use the formula, amount of drug prescribed × weight of the patient in kg × 4 (q6h in a 24-hour period = 4). 3 × 50 = 150 mg to be given every 6 hours. To determine how much the patient will receive in a 24-hour period, divide 6 into 24 to equal 4 doses. 4 × 150 = 600 mg. Cognitive Level: Application Difficulty: Moderate Integrated Process: Nursing process 8. An order is written for oxazepam for a 6-year-old child. The nurse notices that there is no established dosage for children 6 to 12 years of age. Knowing that the usual adult dose is 10 mg t.i.d., what would the nurse calculate the appropriate dose to be? A) 0.03 mg t.i.d. B) 0.3 mg t.i.d. C) 1.8 mg t.i.d.
  • 48. D) 3.3 mg t.i.d. ANSWER: D Response: Because the nurse only knows the child's age, the nurse would need to use Young's Rule to determine the appropriate dosage. Use the formula, child's dose equals the age of the child in years divided by the child's age plus 12 times the average adult dose to calculate the answer. Dose = (6/[6 + 12]) × 10 mg (6/18 = 0.33 × 10 = 3.3). Cognitive Level: Application Difficulty: Moderate Integrated Process: Nursing process 9. An order is written for a patient to receive 1000 mL of normal saline IV over 8 hours. If the IV infusion set is a microdrip set that delivers 60 drops/mL, how many drops
  • 49. Chapter 5: Dosage Calculations per minute should the patient be receiving? A) 60 drops/min B) 120 drops/min C) 240 drops/min D) 480 drops/min ANSWER:B Response: If a patient were to receive 1000 mL in 8 hours, dividing 1000 by 8 would mean that the patient would receive 125 mL in 1 hour, or 60 minutes. Setting up the equation, 60 drops/mL × X = 125 mL/60 min, and cross-multiplying, the answer is 120 drops/min. Cognitive Level: Application Difficulty: Moderate Integrated Process: Nursing process 10.A patient needs to take 1 g PO of cefadroxil. The drug comes in 500-mg tablets. How many tablets should the patient take? A) 0.5 tablet B) 1 tablet C) 2 tablets D) 3 tablets ANSWER: C Response: Convert 1 g to mg by multiplying 1 g times 1000 mg. There are 500 mg in each tablet. Dividing the 1000-mg prescribed dosage by the 500-mg available dosage, the answer is 2 tablets.
  • 50. 1.A nurse is preparing a prescribed dose of rabeprazole for a client with gastric ulcers. The nursing drug guide does not list treatment of gastric ulcers among the recognized indications for this drug. What should the nurse do? A) Administer the drug as ordered B) Clarify with the prescriber concerning the ordered drug C) Document this potential error in the client's health record D) Ask the client whether he or she has taken this drug before 2.The nurse is applying the Centers for Disease Control and Prevention (CDC) recommendations for the possibility of bioterrorism. What action should the nurse perform? A) Post updated information on signs and symptoms of infections caused by biological agents B) Provide guidelines for treating clients exposed to, or potentially exposed to, biological agents C) Remain current on recognition and treatment of infections caused by biological weapons D) Advocate for increased funding for research involving bioterrorism and client treatment 3.A nurse's learning goals for the year include reviewing the most up-to-date information about emergency preparedness related to bioterrorism agents. What should the nurse do? A) Read textbooks devoted to the topic B) Review best practices around triage and emergency care C) Read journal articles about bioterrorism agents D) Visit the Centers for Disease Control and Prevention (CDC) Web site 4.The nurse is assessing a client with diabetes who reports several hypoglycemic episodes during the past 3 weeks. What assessment question best addresses a potential causative factor? A) “Do you ever take St. John's wort?” B) “Have you used any recreational drugs in the past few weeks?”
  • 51. Chapter 5: Dosage Calculations IF YOU WANT THIS TEST BANK OR SOLUTION MANUAL EMAIL ME kevinkariuki227@gmail.com TO RECEIVE ALL CHAPTERS IN PDF FORMAT IF YOU WANT THIS TEST BANK OR SOLUTION MANUAL EMAIL ME kevinkariuki227@gmail.com TO RECEIVE ALL CHAPTERS IN PDF FORMAT